Tải Phương pháp dồn biến chứng minh bất đẳng thức - Tài liệu ôn thi Bất đẳng thức

60 25 0
Tải Phương pháp dồn biến chứng minh bất đẳng thức -  Tài liệu ôn thi Bất đẳng thức

Đang tải... (xem toàn văn)

Tài liệu hạn chế xem trước, để xem đầy đủ mời bạn chọn Tải xuống

Thông tin tài liệu

3) Chuùng toâi nghó laø caùc baïn seõ ñoàng yù raèng: neáu moät baøi toaùn ñaõ chuaån hoùa (töùc laø BÑT coù ñieàu kieän) thì noù seõ "gôïi yù" cho chuùng ta caùch doàn bieán (ph[r]

(1)

PHƯƠNG PHÁP DỒN BIẾN

Phan Thành Việt Nội dung:

1 Giới thiệu

2 BĐT biến với cực trị đạt đối xứng Dồn biến kĩ thuật hàm số

4 BĐT biến với cực trị đạt biên BĐT biến

6 Dồn biến hàm lồi

7 Dồn biến giá trị trung bình Định lý dồn biến tổng quát Nhìn lại

10 Bài tập

1 Giới thiệu.

Các bạn thân mến, nhiều số BĐT mà ta gặp có dấu đẳng thức biến số Một ví dụ kinh điển

Ví dụ 1: (BĐT Cauchy) Cho x, y, z >0 x+y+z ≥3√3xyz

Có thể nói số lượng BĐT nhiều nhiều bạn thấy điều là hiển nhiên Tất nhiên, không hẳn Tuy nhiên,

trường hợp đẳng thức không xảy tất biến ta lại thường rơi vào trường hợp khác, tổng quát hơn: có số (thay tất cả) biến Ở dẫn ví dụ chứng minh phần sau

Ví dụ 2: (VMO) Cho x, y, zR, x2+y2 +z2 = Thì

2(x+y+z)−xyz≤10

(2)

Có thể nhiều bạn ngạc nhiên biết cịn có bất đẳng thức mà dấu "=" xảy biến khác Ví dụ sau chứng minh phần sau

Ví dụ 3: (Jackgarfukel) Cho a, b, c số thực khơng âm có tối đa

một số Thì ta có:

a

a+b+ b

b+c+ c

c+a

5

a+b+c

Ở đây, dấu đẳng thức xảy a= 3b >0, c = 0(và dạng hốn vị) Các bạn tự hỏi giá trị chẳng hạn (3,1,0) có đặc biệt mà làm cho đẳng thức xảy Một cách trực giác, ta thấy dường điểm đặc biệt có biến Vì giả thiết biến khơng âm, nên biến cịn gọi biến có giá trị biên

Tóm lại, BĐT mà ta gặp, có trường hợp dấu "=" xảy thường gặp: trường hợp tất biến (ta gọi "cực trị đạt tâm"), tổng quát trường hợp có số biến (ta gọi "cực trị đạt có tính đối xứng"), trường hợp khác dấu "=" xảy có biến có giá trị biên (và ta gọi "cực trị đạt biên")

Phương pháp dồn biến đặt để giải BĐT có dạng Ý tưởng chung là: ta đưa trường hợp có hai biến nhau, biến có giá trị biên, số biến giảm Do BĐT đơn giản BĐT ban đầu, đặc biệt BĐT biến cách khảo sát hàm biến số ta chứng minh BĐT đơn giản Chính tư tưởng giảm dần số biến nên phương pháp gọi phương pháp dồn biến

(3)

hết sức tổng quát Tinh thần xuyên suốt muốn bạn đọc cảm nhận tính tự nhiên vấn đề Qua đó, bạn lý giải "tại sao", để tự bước đường sáng tạo

*Ghi chú: Chúng tơi đánh dấu tốn theo mục Vì số lượng định lý nên chúng tơi khơng đánh dấu Chúng tơi cố gắng ghi tên tác giả nguồn trích dẫn tất kết quan trọng, ngoại trừ kết

2 BĐT biến với cực trị đạt đối xứng.

Xin phác họa lại tư tưởng sau Bài tốn có dạng f(x, y, z)≥0với x, y, z biến số thực thỏa mãn tính chất

nào Điều mong muốn có đánh giá f(x, y, z)≥f(t, t, z) với t đại lượng thích hợp tùy theo liên hệ x, y, z (ta

gọi kĩ thuật dồn biến nhau) Sau kiểm tra

f(t, t, z) ≥ để hoàn tất chứng minh Lưu ý biến chuẩn hóa bước cuối tốn với biến

Trong mục này, xem xét ví dụ Bài tốn 1. (BĐT Cauchy) Cho x, y, z > 0, chứng minh

x+y+z ≥3√3xyz

Lời giải:

Vì BĐT đồng bậc nên cách chuẩn hóa ta giả sửx+y+z = (*) Viết lại toán dạng f(x, y, z)≥0 với f(x, y, z) = 1−27xyz Ta

thấy thay xy t= x+2y điều kiện (*) bảo tồn (tức có t+t+z= 1), nên ta phải xem xét thay đổi xyz

Theo BĐT Cauchy với biến (chứng minh đơn giản) xyt2,

nên xyzt2z Vậyf(x, y, z)≥f(t, t, z)

Cuối để ý z = 1−2t nên ta có:

f(t, t, z) = 1−27t2z = 1−27t2(1−2t) = (1 + 6t)(1−3t)2 ≥0 toán chứng minh xong Đẳng thức xảy x=y và3t= 1, nghĩa

(4)

*Nhận xét:

1) Có thể nhiều bạn bỡ ngỡ với cách chuẩn hóa Chúng tơi xin nói rõ: khơng có bí ẩn Nếu thích, bạn hồn tồn chuẩn hóa theo cách khác, chẳng hạn giả sử xyz = chứng minh

f(x, y, z)≥0vớif(x, y, z) =x+y+z−3 Khi bước dồn biến chứng

minh f(x, y, z) ≥ f(t, t, z) với t = √xy Đề nghị bạn đọc tự lý giải

trong lời giải ta xét t = x+2y cịn lại xét t = √xy, sau

hồn thành chứng minh theo cách

2) Bạn đọc thắc mắc: khơng cần chuẩn hóa khơng? Câu trả lời là: được! Thật vậy, hồn tồn xét tốnf(x, y, z)≥0 với f(x, y, z) =x+y+z−3√xyz Khi bước dồn biến chứng minh f(x, y, z)≥f(t, t, z)với t = x+2y hay t=√xy Thực chất, điều

hồn tồn dễ hiểu, tương ứng BĐT có điều kiện BĐT khơng điều kiện (qua kĩ thuật chuẩn hóa)

3) Chúng tơi nghĩ bạn đồng ý rằng: tốn chuẩn hóa (tức BĐT có điều kiện) "gợi ý" cho cách dồn biến (phải đảm bảo điều kiện), nhiên, ngược lại tốn chưa chuẩn hóa (BĐT khơng điều kiện) có nhiều cách để dồn biến (nói chung, ta chọn cách dồn biến cho bảo toàn "nhiều" biểu thức BĐT - điều tương đương với chuẩn hóa cho biểu thức có dạng đơn giản nhất) Do đó, phối hợp tốt kĩ thuật chuẩn hóa dồn biến điều cần thiết Tuy nhiên, quen với điều bạn thấy khơng có khác biệt đáng kể chúng

Bài toán 2. (BĐT Schur) Cho a, b, c≥0, chứng minh rằng: a3+b3+c3+ 3abca2(b+c) +b2(c+a) +c2(a+b).

Lời giải:

Xétf(a, b, c) =a3+b3+c3+ 3abca2(b+c)−b2(c+a)−c2(a+b) Đặt t= b+2c, ta hi voïng: f(a, b, c)≥f(a, t, t) Xeùt

d=f(a, b, c)−f(a, t, t) =

h

b+c−5

4a

i

(bc)2

Ta thấy với a, b, c số không âm tùy ý khơng có d ≥0 Tuy

(5)

chứng minh f(a, t, t)≥ Nhưng BĐT tương đương với a(at)2 ≥ 0

nên hiển nhiên Bài toán chứng minh xong

*Nhận xét: Việc giả sử a=min{a, b, c}là thủ thuật thường áp

dụng để dồn biến Nhắc lại BĐT biến đối xứng ta giả sử

abc (hoặc abc), trường hợp BĐT biến hốn vị vịng

quanh ta giả sử a=min{a, b, c} (hoặca =max{a, b, c})

Bài toán 3. Cho a, b, c số thực dương có tích Chứng minh

rằng:

1

a +

1

b +

1

c +

6

a+b+c ≥5.

Hướng dẫn:

Nếu toán ban đầu tốn quen thuộc, tốn khó Với kinh nghiệm thu từ tốn 1, nghĩ tới việc dồn biến theo trung bình nhân để khai thác giả thiết tích ba số Một lời giải theo hướng bạn Yptsoi (Đài Loan) đưa lên diễn đàn Mathlinks, mà sau xin dẫn lại cách vắn tắt

Ta chứng minh f(a, b, c) ≥ f(a,

bc,

bc) giả sử abc

Tiếp theo, ta chứng minh f(a,

bc,

bc)≥5, hay laø f

1

x2, x, x

≥5, với x=

bc

BĐT tương đương với (x−1)2(2x4 + 4x3 −4x2−x+ 2) ≥ Vì biểu

thức ngoặc thứ hai dương với x > 0nên chứng minh hoàn tất Đẳng thức xảy a=b=c=

Qua ví dụ trên, thấy cách dồn biến trung bình cộng trung bình nhân thật hữu dụng Tuy nhiên, cách dồn biến vô phong phú uyển chuyển Ví dụ sau minh họa cho điều Bài tốn 4.(Iran 1996) Chứng minh vớia, b, c >0thì:

(ab+bc+ca)

1

(a+b)2 +

1 (b+c)2 +

1 (c+a)2

4. Hướng dẫn:

(6)

là dấu "=" đạt ngồi a=b=ccịn có a=b, c→0

Các bạn nên thử để thấy cách dồn biến thơng thường trung bình cộng trung bình nhân dẫn đến BĐT vơ phức tạp Lời giải sau lấy từ ý thầy Trần Nam Dũng, mà nhìn kĩ bạn thấy mối tương quan, khơng tính tốn mà tư duy, kĩ thuật chuẩn hóa dồn biến, mà đề cập nhận xét 3) tốn

Vì BĐT đồng bậc nên ta giả sử ab+bc+ca = (*) Bây ta hi vọng có đánh giá f(a, b, c) ≥

4 với f(a, b, c) biểu thức thứ hai

vế trái BĐT cần chứng minh Ở đâyt phải thỏa liên hệ (*), nghĩa t2+ 2tc=

Bằng cách giả sử c = min{a, b, c} ta chứng minh f(a, b, c) ≥ f(t, t, c) Cuối cùng, ta kiểm tra f(t, t, c) ≥

4 Ở bạn đọc thay c= 1−2tt2 vào BĐT để thấy:

f(t, t, c) = (1−t

2)(1−3t2)2

4t2(1 +t2) ≥0

Bài toán chứng minh xong!

*Nhận xét: Ở bước cuối, bạn khơng chuẩn hóa mà quay lại BĐT đồng bậc:

(t2+ 2tc)( (t+c)2 +

1 4t2)≥

9

⇔(t2+ 2tc)(8t2+ (t+c)2)−9(t+c)2t2 ≥0⇔2tc(tc)2 ≥0 Cuối đến với ví dụ mà cực trị khơng đạt tâm, BĐT đối xứng Các bạn thấy rằng, đường phần quan trọng dồn hai biến nhau, cịn sau cực trị đạt tâm hay khơng khơng phải điều mấu chốt

Bài toán 5. (VMO) Cho x, y, z số thực thỏa mãn: x2 +y2 +z2 =

Chứng minh rằng: 2(x+y+z)−xyz≤ 10

Lời giải.

(7)

định ý ta xét t ≥0

Ta có: d = f(x, y, z)−f(x, t, t) = 2(y+z−2t)−x(yzt2) Ta thaáy

ngay y+z−2t ≤0 vàyzt2≤0 Do để cód ≤0 ta cần x≤0

Từ đó, ta giả sử x = min{x, y, z} Xét trường hợp x ≤ Khi

ta dồn biến phải chứng minh f(x, t, t) ≤ 10 Thay t=p(9−x2)/2 ta có:

g(x) = f(x, t, t) = 2x+ 2p2(9−x2)−x(9−x2

)/2 Ta coù:

g0(x) = 3x

2

2 −

5 −

4x

18−2x2

Giải ta thấy phương trình g0(x) = 0 có nghiệm âm làx=−1 Hôn

nữa g0liên tục vàg0(−2)>0> g(0) nên suy rag0 đổi dấu từ dương sang âm

khi qua điểm x =−1 Vậy ∀xg(x) ≤g(−1) = 10 ta có điều phải chứng minh Trường hợp đẳng thức đạt tạix=−1, y =z=

Phần lại ta phải giải trường hợp x >0, tức số x, y, z

dương Lúc dấu BĐT thực ta cần đánh giá đơn giản thông qua dồn biến Nếu x≥3/4

f(x, y, z) = 2(x+y+z)−xyz≤2p3(x2 +y2+z2)−(3

4)

3

=

27−27

64 <10 Nếu x≤3/4

f(x, y, z) = 2(x+y+z)−xyz≤2(p2(y2+z2)+ 3/4) ≤= 2(√18 + 3/4) <10

Bài toán chứng minh xong!

3 Dồn biến kó thuật hàm số.

(8)

Trong $2 thấy để chứng tỏ f(x, y, z) ≥ f(t, t, z) ta việc xét hiệu d =f(x, y, z)−f(t, t, z)rồi tìm cách đánh giá cho d ≥0

Tuy nhiên, dạng BĐT đơn giản, phù hợp với biến đổi đại số Giả sử ta phải làm việc với biểu thức f có dạng, chẳng hạn, như: f(x, y, z) = xk +yk +zk với k > 0 cách biến đổi đại số trở nên cồng kềnh phức tạp

Kĩ thuật hàm số dùng để giải trường hợp Ý tưởng này, chẳng hạn để chứng minh f(x, y, z) ≥ f(x, t, t) với t = (y+z)/2, ta xét hàm: g(s) =f(x, t+s, ts)vớis ≥0 Sau chứng minh g tăng với s ≥ (thông thường dùng công cụ đạo hàm tiện lợi), suy

g(s) ≥ g(0), ∀s ≥ 0, ta thu điều mong muốn Một

ví dụ quen thuộc với bạn dồn biến hàm lồi, nhiên quan sát kĩ thuật dồn biến bối cảnh tổng quát hơn, vấn đề hàm lồi trở lại mục sau toán với n biến

Chúng nhấn mạnh rằng, kĩ thuật khó, chứa đựng nét tinh tế phương pháp dồn biến Những ví dụ sau thể rõ vẻ đẹp sức mạnh phương pháp dồn biến

Bài toán 1. Cho k > a, b, c số khơng âm có tối đa

số Chứng minh rằng: ( a

b+c)

k

+ ( b

c+a)

k

+ ( c

a+b)

k

min{2,

2k} (∗) Lời giải:

Tất nhiên ta cần chứng minh BĐT = 23k ⇔ k =

ln3

ln2 −1 (caùc

bạn suy nghĩ BĐT cho trường hợp lại dẫn đến BĐT cho trường hợp tổng quát) Chú ý với k đẳng thức xảy

ra hai chỗ a=b=c a =b, c= (và hoán vị)

Khơng tổng qt giả sử a+b+c = bca Đặt t = b+2cm = b−2c, suy b=t+m, c=tm, a= 1−2t Khi vế trái

BĐT cần chứng minh là:

f(m) =

1−2t

2t

k +

t+m

1−tm

k +

tm

1 +mt

kca nên 3t−1≥m≥0, 1≥b+c= 2t nên 12 ≥t

3

(9)

Ta coù:

f0(m) = k(t+m) k−1

(1−tm)k+1 −

k(tm)k−1 (1 +mt)k+1 f0(m)≥0⇔ (t+m)

k−1

(1−tm)k+1 ≥

(tm)k−1

(1 +mt)k+1 ⇔g(m) := [ln(tm)−ln(t+m)]− k+

1−k [ln(1−tm)−ln(1 +mt)]≥0

Tiếp tục khảo sát g, ta coù: g0(m) =−

1

tm +

1

t+m

+ k+ 1−k

1

1−tm +

1 +mt

≥0

⇔ −2t

(tm)(t+m)+

k+ 1−k.

2(1−t)

(1−tm)(1 +mt) ≥0 (1) Đánh giá k+1

1−k ≥2, để chứng minh (1) ta cần chứng minh

⇔ −t

t2−m2 +

2(1−t)

(1−t)2−m2 ≥0 (1) ⇔u(m) =−t+ 4t2 −3t3+ 3tm2−2m2 ≥0

Thật vậy, u0(m)<0 nên u(m)≥ u(3t−1) = 2(3t−1)(2t−1)2 ≥0 Vậy g(m) đồng biến suy g(m) ≥ g(0) = suy f0(m) ≥ suy

f(m)≥f(0) Nhớ m = b=c=t

Cuối cùng, ta cần chứng minh h(t) :=f(0) ≥2 Viết lại: h(t) =

1−2t

2t

k +

t

1−t

k

Ta khaûo sát h(t)trên miền t∈[0,13] Ta có: h0(t) = 2kt

k−1

(1−t)k+1 − k

2k.

(1−2t)k−1 tk+1 ≤0 ⇔2k+1t2k ≤[(1−t)(1−2t)]k−1 (2)

Trong BĐT cuối, vế trái hàm đồng biến theot vế phải hàm nghịch

biến theo t, lưu ý t

3 nên để chứng minh (2) ta cần:

2k+1

1

2k

≤[(1−

3)(1− 3)]

(10)

Bất đẳng thức đúng, nên h(t) nghịch biến, suy

h(t)≥h(1 3) = Bài toán giải trọn vẹn!

Nhận xét: Để thấy nét đẹp tốn này, chúng tơi xin dẫn số trường hợp riêng nó, thân chúng toán hay biết đến cách rộng rãi

1) Trường hợp k = 1, ta thu BĐT Netbit: a

b+c+ b c+a +

c a+b

3

Đây BĐT tiếng Một cách chứng minh "kinh điển" là:

a b+c +

b c+a +

c

a+b + =

a+b+c

b+c +

a+b+c

a+c +

a+b+c a+b

= (a+b+c)(

b+c+

1

c+a +

1

a+b)

≥(a+b+c)

(b+c) + (c+a) + (a+b) =

2) Trường hợp k = 12, ta thu BĐT sau:

r

a b+c +

r

b c+a +

r

c a+b ≥2

Đây toán đẹp, trước biết đến BĐT ngược chiều với BĐT Netbit Có lời giải đơn gản, dùng BĐT

Cauchy: r

a b+c =

2a

2pa(b+c)

≥ 2a

a+b+c

3) Trường hợp k

3, ta có BĐT sau:

( a

b+c)

k

+ ( a

b+c)

k

+ ( a

b+c)

(11)

Đây toán đẹp biết đến từ trước mở rộng cho BĐT Netbit (nó đăng tạp chí THTT với tên tác giả Trần Tuấn Anh) Từ kết toán tổng quát, ta biết 2/3 số tốt để có giá trị nhỏ 3/2k Tuy nhiên, số tốt theo nghĩa áp dụng BĐT Cauchy theo cách sau Để đơn giản chúng tơi trình bày với trường hợp k = 2/3

a+b+c=a+b+c

2 +

b+c

2 ≥3

3

r

a(b+c )

2

⇒( 2a

b+c)

2

3 ≥ 3a

a+b+c

Cùng với toán 1, toán sau ví dụ đẹp cho kĩ thuật hàm số

Bài toán 2. Cho k >0, a, b, c≥0 vàa+b+c= Chứng minh rằng:

(ab)k+ (bc)k+ (ca)kmax{3,(3 2)

k} (∗) Lời giải:

Không tổng quát giả sử bc (cịn việc cho a = min hay max tùy theo tình huống, ta điều chỉnh cách "hợp lí" cần

thieát)

Đặtt = b+2cm= b−2c suy b=t+m, c=tm Khi vế trái BĐT

cần chứng minh trở thành:

f(m) =ak[(t+m)k+ (tm)k] + (t2−m2)k Ta khảo sát f(m) miền m ∈[0, t] Ta coù:

f0(m) =kak[(t+m)k−1−(tm)k−1]−2km(t2−m2)k−1

f0(m)≥0⇔g(m) :=ak[(tm)1−k −(t+m)1−k]−2m ≥0

Tất nhiên ta cần xét k > (khi k ≤ tốn đơn giản) Ta có:

(12)

g0(m) đồng biến, có tối đa nghiệm (0, t) Vì g(0) =

0, g(t) = +∞ nên có hai khả năng:

g(m)>0 g(m) =− +

Tương ứng ta cóf(m)đi lên hoặcf(m)đi xuống lại lên Trong trường hợp cực đại đạt biên

f(m)≤max{f(0), f(t)}

Nhắc lại m = 0⇔b=c=tm=tc=

Dễ thấy c= thì:

f(t) = 2(ab)k

3

2k nên ta phải xét trường hợp lại Đặt:

h(t) :=f(0) = 2tkak+t2k = 2tk(3−2t)k +t2k

Ta coù:

h0(t) = −4k(3−2t)k−1tk+ 2k(3−2t)kbk−1+ 2kb2k−1 h0(t)≥0⇔ −2

3−2t t

k−1

+

3−2t t

k

+ ≥0

u(x) :=xk −2xk−1+ ≥0 với x= 3−2t

t

Ta coù: u0(x) = [kx−2(k−1)]xk−2.u0(x) có tối đa nghiệm R+

nên u(x) có tối đa2 nghiệm R+, nghiệm x=

Từ đó, ta giả sử a=min{a, b, c} Khi ta việc xét t ≥1 tương ứng x≤1 Vìu(x) có tối đa nghiệm (0,1) nên h0(t) có tối đa nghiệm (1,32)

Lưu ý lưu ýh0(1) = 0, h0(3

2)>0 Do đó, có hai khả hoặch(t)

đồng biến h(t) có dạng−0+ Trong trường hợp thìh(t)cũng đạt max hai biên, suy ra:

h(t)≤max{f(1), f(3

2)}=max{3,( 2)

(13)

*Nhận xét: Ở không giả thiết a=min{a, b, c}ngay từ đầu

muốn nhấn mạnh rằng: việc dồn biến thực mà không cần thứ tự biến Tận dụng điều đó, làm cách khác để né việc khảo sát toán biến

Thật vậy, chứng minh ra, ta ln có BĐT sau mà khơng cần giả thiết thứ tự a, b, c:

f(a, b, c)≤ max{(3 2)

2k

, f(a,b+c

2 ,

b+c

2 )} (∗)

Từ đó, với a, b, c cố định, xét dãy số sau: (a0, b0, c0) = (a, b, c), ∀nZ+ ta định nghĩa quy nạp:

(a2n−1, b2n−1, c2n−1) = (a2n−2,

b2n−2+b2n−2

2 ,

b2n−2+b2n−2

2 )

vaø:

(a2n, b2n, c2n) = (

a2n−1+b2n−1

2 ,

a2n−1+b2n−1

2 , c2n−1) ta có

f(a, b, c)≤max{(3 2)

2k

, f(an, bn, cn)},nZ+

Dễ thấy dãy {an}, {bn}, {bn} hội tụ 1, nên chuyển qua giới hạn ta có điều phải chứng minh

Kĩ thuật chuyển qua giới hạn tự nhiên Nó tổng quát lên thành định lý dồn biến tổng quát SMV UMV mà giới thiệu phần sau Cũng sử dụng tính liên tục hàm số với kĩ thuật khác, chúng tơi cịn đạt kết tổng qt

Sau có (*), cịn cách khác để đạt điều phải chứng minh mà cần sử dụng số hữu hạn lần thay Tuy nhiên, để khỏi trùng lắp giới thiệu mục BĐT biến (và mục sau), mà thực cần thiết

h Cịn trường hợp biến, chúng tơi sử dụng cách tiếp cận đơn giản (dồn biến khảo sát), nhằm giữ tính sáng tư tưởng

(14)

thiết dồn trung bình cộng Sau ví dụ cho kiểu dồn biến trung bình nhân

Bài toán 3: (Phạm Kim Hùng)

a)Cho số thực dương a, b, c có tích Chứng minh rằng: (i) 81(1 +a2)(1 +b2)(1 +c2)≤8(a+b+c)4

(ii) 64(1 +a3)(1 +b3)(1 +c3)≤(a+b+c)6

Lời giải:

(i) Đặt f(a, b, c) = 8(a+b+c)4−81(1 +a2)(1 +b2)(1 +c2) Ta giaû

sử ab Xét hàm số g(t) =f(ta, b/t, c)với t ∈[pb/a,1] Ta có: g0(t) = 32(ab

t2)(ta+ b t +c)

3−

81(ab

t2)(ta+ b

t)(1 +c

) Vì t ∈[pb/a,1]nên g0(t)≥0nếu:

32(d+c)3 ≥81d(1 +c2) với d=ta+ b

t

Ta coù: 32(d+c)3 >32d(d2+ 2dc+ 3c2)≥32d(3

d4c2+ 3c2

)>81d(1 +c2) (lưu ý d2c≥4)

Vậy g0(t)≥0vớit ∈[pb/a,1] Do đó: g(1)≥g(pb/a) Vậy f(a, b, c)≥ f(s, s, c)với s=

ab Thays = 1/cta được: f(s, s, c) =f(√1

c,

1

c, c) = 8(

2

c+c)

4−

81(1 +

c)

(1 +c2) = (

c−1

c )

2

(8c5+ 16c92 + 24c4+ 96c

2 + 87c3+ 78c 2+

+99c2+ 120c32 −21c+ 94

c+ 47)

≥0 (ñpcm)

Đẳng thức xảy a=b=c=

(15)

toán phức tạp Rồi thành cơng trước tốn khiến bạn tự tin Chúng dẫn tốn mà lời giải khiến nhiều bạn "khiếp sợ", nhiên hi vọng bạn bình tâm để thấy vẻ đẹp sáng ẩn đằng sau kĩ thuật tính tốn lão luyện

Bài tốn 4. Cho a, b, c ≥ 0, a +b +c = Tìm giá trị lớn

biểu thức:

ab

3 +c2 + bc

3 +a2 + ca

3 +b2

Lời giải:

Lời giải sau anh Phan Thành Nam

Giả sử abc Đặt a = s+t, b = st vế trái BĐT cần chứng

minh laø:

f(t) := c(st) + (s+t)2 +

c(s+t) + (st)2 +

s2−t2

3 +c2

Ta khảo sát f(t) miền t ∈[0, sc] Ta coù: f0(t) = −c

3 + (s+t)2 −

2c(s2 −t2)

(3 + (s+t)2)2 +

c

3 + (st)2 +

2c(s2−t2)

(3 + (st)2)2 −

2t

3 +c2

= 4cst

uv +

8cst(s2−t2)(u+v)

u2v2 −

2t

3 +c2 <0,t ∈(0, sc) (∗)

với u= + (s+t)2, v= + (st)2 (BĐT (*) chứng minh sau)

Vậy ∀t ∈[0, sc] thì:

f(t)≤f(0) = 2cs +s2 +

s2

3 +c2 =

2s(3−2s) +s2 +

s2

3 + (3−2s)2 =:g(s) (1)

Xét g(s) với s∈[1,32] Ta có: g0(s) = 24s−12s

2

(3 + (3−2s)2)2+

18−24s−6s2

(3 +s2)2 =

108(s2 −3s+ 4)(s−1)2(−s2−3s+ 6)

[3 + (3−2s)2]2[3 + s2]2

Dễ thấy s2−3s+ 4>0và−s2−3s+ = (

33−3

2 −s)(s+

33+3

2 ) neân g

0(s)

dương (1, s0) âm (s0,32) với s0 :=

33−3

2 = 1,372281323

Vaäy ∀s ∈[1,3 2] thì:

g(s)≤g(s0) = 11

33−45

(16)

Trong (1) (2), dấu "=" xảy đồng thời t = s = s0, tức a=b=s0c= 3−2s0

Vậy giá trị lớn cần tìm 11√33−45

24 = 0,757924546 , đạt a=b=

33−3

2 = 1,372281323 …, c= 6− √

33 = 0,255437353

Để kết thúc, ta chứng minh BĐT (*) Đây BĐT chặt Ta với t∈(0, sc) thì:

4cs uv <

1

3 +c2 (3) vaø

8cs(s2−t2)(u+v)

u2v2 ≤

1

3 +c2 (4)

laø xong!

Chứng minh (3): Vì c+ 2s = s > nên cs < Hơn u = + (s+t)2 >4, v= + (st)2 >3 +c2 Từ suy ra(3)

Chứng minh (4): Dùng BĐT Cauchy ta có:

u2v2 = [[3 + (s + t)]2[3 + (s - t)]2]2 ≥16(s2 −t2),vaø

2cs(u+v)(3 +c2) = 4cs(3 +s2+t2)(3 +c2)≤

4cs+ +s2+t2+ +c2

3

3

Thay c= 3−2s vào, lưu ý tsc= 3s−3, ta có:

4cs+ +s2+t2+ +c2 ≤4(3−2s)s+ +s2+ (3s−3)2+ (3−2s)2 = 12 + 6(s−1)(s−2) ≤12

suy 2cs(u+v)(3 +c2)<43 Vaäy: 8cs(s2−t2)(u+v)

u2v2 = 4.

s2−t2 u2v2 .

2cs(u+v)(3 +c2)

3 +c2 ≤4.

1 44.

43

3 +c2 =

1 +c2

và toán giải xong!

4 BĐT biến với cực trị đạt biên.

Nếu phần trước hiểu "dồn biến" "đẩy hai biến lại gần nhau", trường hợp ta phải hiểu "dồn biến" nghĩa "đẩy biến biên" Chẳng hạn xét BĐT f(x, y, z) ≥ với x, y, z ≥ 0, ta hi vọng vào đánh giá f(x, y, z) ≥ f(0, s, t),

(17)

là kĩ thuật dồn biến biên) Tất nhiên ta chọn s, t cho hiệu d = f(x, y, z) ≥ f(0, s, t) đơn giản đánh giá thuận lợi Cuối ta việc kiểm chứng f(0, s, t)≥0

Trước hết, để bạn làm quen với cách dồn biến "mới mẻ" này, xin trở lại ví dụ phần trước

Bài tốn 1: (BĐT Schur) Cho a, b, c≥0 Chứng minh rằng: a3+b3+c3+ 3abca2(b+c) +b2(c+a) +c2(a+b).

Lời giải:

Trong$2, giải cách dồn biến Tuy nhiên nhận xét điểm a =b=c, đẳng thức đạt a=b, c= (và hốn vị) Do đó, kĩ thuật dồn biến biên có khả thành cơng!

Đặt f(a, b, c) = a3+b3+c3+ 3abca2(b+c)−b2(c+a)−c2(a+b)

Ta hi vọng có f(a, b, c)≥f(0, a+b, c) Xét hiệu: d=f(a, b, c)−f(0, a+b, c) =ab(5c−4a−4b)

Như ta có d ≥0, cho dù tận dụng kiện a, b, c

thể

Thật đáng tiếc! Tuy nhiên, bạn dừng lại cịn đáng tiếc Thay bỏ dỡ, ta xem lại khơng thể có d ≥ Nếu

tinh ý, bạn thấy f(a, b, c) nhỏ hai biến tiến lại gần (đó lý mà ta dồn hai biến $2), thay bộ(a, b, c)bởi (0, a+b, c)thì "dường như" biến cách xa Đó lý cách dồn biến thất bại

Từ đó, ta nảy ý thay (a, b, c)bởi (0, b+a/2, c+a/2) Xét hiệu: da=f(a, b, c)−f(0, b+a/2, c+a/2) =a(a+b−2c)(a+c−2b) Điều thú vị ta giả sửda ≥0 Thật vậy, điều nhờ việc thứ tự biến a, b, cmà hiệuda, db, dc (trong đódb, dc hai hiệu tương tự nhưda) Vì tính đối xứng nên ta có thể giả sử da =max{da, db, dc} Khi da <0

(18)

và mâu thuẫn!

Vậy da ≥ nên f(a, b, c)≥ f(0, s, t) với s =b+a/2, t =c+a/2 Cuối cùng, ta thấy

f(0, s, t) =t3+s3−t2sts2 = (t+s)(ts)2 ≥0 chứng minh hoàn tất

*Nhận xét: Mặc dù BĐT Schur quen thuộc, cách chứng minh dồn biến ý gần Tuy nhiên, cách dồn hai biến "hợp lý", cách dồn biến biên kết thực bất ngờ Tất nhiên, chứng minh cách ngắn gọn nhất, muốn nhấn mạnh đến tự nhiên Nếu toán việc áp dụng kĩ thuật dồn biến biên gây bất ngờ, tốn sau đường tất yếu

Bài toán 2: (Hojoo Lee) Cho a, b, c ≥ 0, ab +bc + ca = (*) Chứng minh rằng:

1

a+b +

1

b+c +

1

c+a

5 Lời giải:

Bài đẳng thức không xảy tâm, mà a =b= 1, c = hoán vị Xét trường hợp riêng c= 0, toán trở thành:

"Chứng minh rằng:

a +

1

b +

1

a+b

5

2,với ab= 1.” Đặts=a+bthì điều tương đương vớis+1s

2, hay(2s−1)(s−2)≥

0 BĐT cuối hiển nhiên s=a+b≥2

ab=

Vậy ta cần dồn biến xong Cách làm sau lấy từ ý anh Phạm Kim Hùng Diễn Đàn Tốn Học

(19)

hiệu:

d=f(a, b, c)−f(a+b, a+b,0)

=

a+b+

1

a+ 1a−+abb +

b+1a−+abb

!

a+b+a+b+

1

a+b+ a+1b

!

.

=

1 +a2 +

1

1 +b2 −1−

1 + (a+b)2

Từ quy đồng lên ta thấy d ≥ 2(1−ab) ≥ ab(a+b)2 Nếu giả sử

c =max{a, b, c} 2(1−ab) = 2c(a+b) ≥ ab(a+b)2 Vậy lúc d ≥0 toán chứng minh xong!

*Nhận xét:

1) Lời giải đầy tiên đưa Diễn Đàn Toán Học anh Phan Thành Nam, cách chứng minh ngắn gọn Đặt x=a+b+c

Nếu x≥2 thì:

a+b =c+ ab

a+bc+ ab

a+b+cf(a, b, c)≥ x+

1

x

5 Nếu x≤2 giả sử a=max{a, b, c}ta có:

f(a, b, c) = (c+ ab

a+b) + (b+ ac a+c) +

1

b+c

= (b+c+

b+c) +

a(1 +bc)

ax+bc ≥2 +

1 =

5 (lưu ý 2a(1 +bc) = 2a+ 2abcax+bc, x≤2vaø 2a≥1 )

Tuy nhiên, lời giải dễ dàng nghĩ Về lời giải dồn biến trên, lần nhấn mạnh đến tính tự nhiên

2) Bài toán toán hay thu quan tâm nhiều bạn Tuy nhiên, bạn bất ngờ hệ đơn giản BĐT quen thuộc khác Đó BĐT Iran 1996 Thật vậy, với giả thiết

ab+bc+ca= từ kết BĐT Iran 1996 ta có ngay: (

a+b+

1

b+c+

1

c+a)

=

(a+b)2 +

1 (b+c)2 +

1 (c+a)2 +

4(a+b+c)

(a+b)(b+c)(c+a) ≥

4 + = 25

(20)

(lưu ý a+b+c= (a+b+c)(ab+bc+ca)≥(a+b)(b+c)(c+a))

Từ nhận xét trên, ta nhớ lại $2, BĐT Iran 1996 giải kĩ thuật dồn hai biến Từ có hai câu hỏi tự nhiên là, thứ nhất: tốn giải cách dồn hai biến không, thứ hai: BĐT Iran 1996 giải cách dồn biến biên không? Chúng đề nghị bạn tự giải đáp hai câu hỏi

3) Bài tốn lại dẫn đến kết thú vị sau đây, mà tác giả bạn Zhao bin (Trung Quốc)

"Cho x, y, z số thực không âm có tối đa số Chứng

minh raèng:

1

x2+y2 +

1

y2+z2 +

1

z2+x2 ≥

10

(x+y+z)2.

Bằng hai toán "cũ" trên, chúng tơi muốn bạn đọc có cảm giác dễ dàng kĩ thuật dồn biến biên Tuy nhiên, hai kĩ thuật dồn bai biến phát huy tác dụng, khơng khỏi khó khăn việc thuyết phục bạn đọc sức mạnh kĩ thuật dồn biến biên Do đó, chúng tơi dẫn tốn sau đây, bạn thấy kĩ thuật dồn hai biến hồn tồn bế tắc, đơn giản đẳng thức đạt biến đôi khác Đây

trong ví dụ quan trọng kĩ thuật dồn biến biên mà chúng tơi muốn trình bày với bạn

Bài toán 3: (Jackgarfukel) Cho a, b, c số thực khơng âm có tối

đa số Chứng minh rằng:

a

a+b + b

b+c + c

c+a

5

a+b+c (∗) Lời giải:

Trước công này, ta cần xem trường hợp dấu xảy ra: dễ thấy a=b=ckhơng thỏa, cách tự nhiên ta nghĩ đến

trường hợp biên: c= Với c= BĐT(*) trở thành

a

a+b+ √

b

4

(21)

Chuẩn hóa a+b= Ta có

(1)⇔1−b+

b

4 ⇔(

b−1/2)2 ≥0 (đúng!) Vậy đẳng thức xảy a= 3b, c= (và hoán vị)

Như trường hợp dấu xảy ba biến rời nhau, phương pháp dồn hai biến xem khơng cịn tác dụng Do đó, dồn biến biên xem đường tất yếu

Không tổng quát giả sử a = max{a, b, c} a+b+c =

Đặt t= a+2c s= a−2c, suy raa =t+s, c=ts, b= 1−2t Ta coù

(∗)⇔ √ t+s s+ 1−t +

1−2t

1−ts + ts

2t

5

4 (1)

Đặtf(s) =V T(1)vớis ∈[0, t], ta chứng minhf(s)≤max(f(0), f(t))

Ta coù :

f0(s) = √

s+ 1−t

t+s

2(s+ 1−t)3/2 +

1−2t

2(1−ts)3/2 −

1

2t

Vì chưa xác định dấu f0(s) nên ta đạo hàm tiếp

f00(s) =−

(s+ 1−t)3/2 +

3(t+s) 4(s+ 1−t)5/2 +

3(1−2t) 4(1−ts)5/2 f000(s) =

4(s+ 1−t)5/2 −

15(t+s) 8(s+ 1−t)7/2 +

15(1−2t) 8(1−ts)7/2

= 18 + 3s−33t (1−ts)7/2 +

15(1−2t)

8(1−ts)7/2 >0, vìb= 1−2t ≥0

Vậy f000(s) > với s ∈ [0, t] nên theo định lí Rolle ruy f0(s) có tối đa đa hai nghiệm [0, t] Mặt khác dễ dàng chứng minh f0(0) ≤ f0(t) ≥ 0 f0(s) đổi dấu tối đa lần (0, t),

nữa f0(s) có dạng sau: f0(s)> 0,s ∈ (0, t)

f0(s)<0,s∈(0, t) f0(s) có dạng − 0 + (0, t). Tuy nhiên

trường hợp f(s)cũng đạt cực đại biên

Vậy f(s)≤max(f(0), f(t))với mọis∈[0, t]nên ta cần chứng minh BĐT sau xong:

(22)

Muốn ta chứng minh BĐT f(0) ≤ 5/4 f(t) ≤ 5/4

Việc chứng minh hai BĐT dễ dàng, nên đề nghị bạn đọc tự kiểm chứng

Hẳn nhiên bạn đồng ý cần thiết phương pháp dồn biến biên toán Tuy nhiên, nhiều bạn cho rằng: tốn khơng đối xứng nên khơng xảy trường hợp dấu "=" có hai biến Để phủ định nhận xét đó, chúng tơi kết thúc mục cách dẫn toán anh Phạm Kim Hùng THTT: Bài toán 4. Cho a, b, c≥0, a+b+c= Chứng minh rằng:

(a3+b3+c3)(a3b3+b3c3+c3a3)≤ 36(ab+bc+ca) Lời giải:

Khơng tổng qt giả sử abc Đặt

f(a, b, c) = 36(ab+bc+ca)−(a3+b3+c3)(a3b3+b3+c3+c3a3) Khi f(a, b+c,0) = 36a(b+c)−(a3+ (b+c)3)a3(b+c)3

Ta chứng minh f(a, b, c)≥f(a, b+c,0).Thật vậy, ý rằng:

36(ab+bc+ca)≥36a(b+c)

(a3+b3+c3)(a3b3+b3c3+c3a3)≤[a3+ (b+c)3]a3(b+c)3 (vì ta có a3+b3+c3 ≤a3+ (b+c)3 vàa3b3+b3c3+c3a3 ≤a3(b+c)3)

Do ta cần chứng minh toán trường hợp c= 0, hay

36aba3b3(a3+b3)⇔36 ≥a2b2(a3 +b3)

Đặt t =ab, bất đẳng thức viết lại dạng t2(27−9t)≤ 36 ⇔ t3+ 4 ≥ 3t2 Nhưng lại BĐT Cauchy ba số t3/2, t3/2,4 Đẳng

(23)

*Nhận xét: Một ví dụ nữa, đơn giản hơn, tác giả Diễn Đàn Toán Học:

" Cho a, b, c≥0, a+b+c= Tìm giá trị lớn của:

(a2 −ab+b2)(b2 −bc+c2)(c2−ca+a2).” Bài toán khơng khó đề nghị bạn đọc tự giải

5 BĐT biến.

Sau nắm vững kĩ thuật dồn biến với số bạn đọc mục cách nhanh chóng Chúng xin lưu ý đặc thù trường hợp biến: Khi có biến ta dồn biến theo cặp, chứng minh toán (chẳng hạn BĐT Cauchy) Tuy nhiên, thuận lợi thường xuất toán đơn giản Đối với phức tạp thường ta dồn cặp nhờ thứ tự biến Sau dồn hai biến (hoặc dồn biến biên) ta chưa có BĐT với biến, mà phải qua BĐT trung gian (2 hay biến) Tuy nhiên thường BĐT trung gian dễ để chứng minh trực tiếp đánh giá để quy biến Nói chung, nhấn mạnh điều cần thiết bạn cần quan sát thật kĩ mối liên hệ biến để có cách xử lý thích hợp

Chúng ta bắt đầu với ví dụ "kinh điển" cho kĩ thuật dồn biến với BĐT biến

Bài toán 1. (IMO SL, Việt Nam đề nghị) Choa, b, c, d≥0, a+b+c+d=

Chứng minh rằng:

abc+bcd+cda+dab

27 + 176

27 abcd Lời giải:

Bài đẳng thức xảy a = b = c = d = 1/4 a = b = c = 1/3, c = Do đó, đánh giá thơng thường dễ rơi vào bế tắc

Đặt f(a, b, c, d) = abc+bcd+cda+dabkabcd với k= 176

(24)

Từ đó, ta hi vọng có f(a, b, c, d)≤f(t, t, c, d)vớit= a+2b Vì0≤abt2

nên để có điều ta cần c+dkcd ≥ Ở may mắn có

điều có điều ngược lại, nghĩa c+dkcd <0, BĐT ban đầu hiển

nhiên vì:

f(a, b, c, d) =ab(c+dkcd)+cd(a+b)≤cd(a+b)≤(c+d+ (a+b)

3 )

3

= 27 Vậy ta giả sử ln có f(a, b, c, d)≤f(a+2b,a+2b, c, d) Lưu ý ta

đã thực việc dồn biến mà không cần giả thiết phụ áp đặt lên biến a, b Do nhờ tính đối xứng ta dồn

biến biến Từ đó, đặt thêm s= c+2d ta có:

f(a, b, c, d)≤f(t, t, c, d)≤f(t, t, s, s) =f(t, s, t, s)

f(t+s ,

t+s

2 , t, s)≤f(

t+s

2 ,

t+s

2 ,

t+s

2 ,

t+s

2 ) =f( 4,

1 4,

1 4,

1 4) =

1 27 toán chứng minh xong!

*Nhận xét:

1) Trong lời giải trên, thực chất bước ta lại phân trường hợp: có trường hợp dồn biến trường hợp mà BĐT hiển nhiên Do đó, lời giải khơng khỏi có phần rối rắm Bạn đọc nên trình bày lại cách phản chứng (giả sử có (a0, b0, c0, d0)sao cho f(a0, b0, c0, d0)> 271)

sẽ gọn gàng chặt chẽ Một cách khác gộp hai trường hợp lại:

f(a, b, c, d)≤max{

27, f(

a+b

2 ,

a+b

2 , c, d)} (1)

2) Ở cịn có cách nhìn nữa, nhìn khơng khác ý (thậm chí dài dịng hơn), nhiên kĩ thuật có ích Ý tưởng lấy từ anh Phan Thành Nam anh Phạm Kim Hùng Diễn Đàn Mathlinks

Nhắc lại f(a, b, c, d) =ab(c+dkcd) +cd(a+b) Đặt g(x) =ab(c+

dkcd) +cd(a+b) g hàm tuyến tính, ab∈[0, t2] (vớit = a+b

2 ) neân g(ab)≤max{g(0), g(t2)} Chú ý g(0) =f(0, a+b, c, d) Vậy ta có:

(25)

Với cách viết BĐT(2) việc cực trị đạt tâm biên rõ ràng Thật ra, toán ta có f(0, a+b, c, d) ≤

27

và chuyển (2) (1) Tuy nhiên, với phức tạp dạng (2) tỏ có ích, đặc biệt kĩ thuật dồn biến tổng quát cho n số mà

chúng tơi trình bày phần sau

3) Các bạn tự giải toán tương tự sau Nguyễn Anh Cường

"Giả sử x, y, z, t số thực không âm thỏa mãn x+y+z +t = 4,

chứng minh rằng:

3(x2+y2+z2+t2) + 4xyzt≥16 ”.

Chúng ta tiếp tục với tốn mà kĩ thuật dồn biến thực rõ ràng

Bài toán 2. (Phan Thành Nam) Cho a, b, c, d số thực khơng âm

có tổng Chứng minh bằng:

abc+bcd+cda+dab+ (abc)2+ (bcd)2+ (cda)2+ (dab)2≤ Lời giải:

Lời giải sau tác giả toán Đặt f(a, b, c, d) V T BĐT cần

chứng minh Ta có:

f(a+b ,

a+b

2 , c, d)−f(a, b, c, d) = (ab

2 )

2

(c+d) + (ab )

2

[(a+b )

2

+ab](c2+d2)−(ab)

2 c

2 d2 ≥(ab

2 )

2

(c+d+ 4abcd−2c2d2)

Vậy c+d+ 4abcd ≥2c2d2 f(a+2b,a+2b, c, d)≥f(a, b, c, d)

Ta giả sử abcdthì theo ta có: f(x, x, c, d)≥f(a, b, c, d)với

x= a+2b Tương tự, ta xét: f(x, x,c+2d,c+2d)−f(x, x, c, d)

Neáu 2x+ 4x2cd≥ 2x4 f(x, x,c+d

2 ,

c+d

2 ) ≥f(x, x, c, d) Và ta cần

chứng minh f(x, x, y, y)≤8 với x+y = Điều đơn giản

(26)

vaø x2(c+d) ≤ (4/3)3 neân f(x, x, c, d) <= (4/3)3 + (4/3)6 < Bài

tốn chứng minh xong! *Nhận xét:

1) Về điều kiện c+d + 4abcd ≥ 2c2d2 để dồn hai biến a, b nhau, ta

thấy cần abcd đủ Điều có nghĩa giả sử abcd

thì ta dồn hai biến biến a, b, c (hơn

nếu biến chưa BĐT thực sự, nghĩa sau dồn biến hàm f tăng lên đại lượng > 0) Liệu điều có dẫn đến: f(a, b, c, d)≤f(t, t, t, c) với t= a+3b+c hay không?

Rõ ràng, giả sử f đạt cực đại (a, b, c, d) theo ta phải có

a = b=c Trên Diễn Đàn Mathlinks bạn Zhao Bin có lời giải với ý

tưởng Tuy nhiên, việc tồn cực đại hàm f (với biến)

là chuyện hiển nhiên (mặc dù rõ ràng mặc trực giác)

Một ý nữa, cách dồn biến liên tiếp biến a, b, c ta

dùng dãy số để chuyển qua giới hạn đưa biến Nhưng lần nữa, rõ ràng mặc trực giác cách làm không phù hợp với cách tiếp cận sơ cấp

Tuy nhiên, tốn bên chúng tơi cung cấp cho bạn cách làm thú vị để chuyển biến trường hợp

2) Nói thêm tốn Bài khơng khó theo lời tác giả tốn đặt để giải toán sau anh Phạm Kim Hùng: "Chứng minh với số không âm a, b, c, d có tổng thì:

1 5−abc +

1 5−bcd +

1 5−cda +

1

5−dab ≤1.

bằng cách sử dụng bổ đề sau đây: "Cho số xi ≥0thỏa mãn:

P4

i=1(xi+x2i)≤8 vàxi+xj ≤3 ,∀i6=j Thì:

5−x1 +

1 5−x2 +

1 5−x3 +

1

5−x4 ≤1.

(27)

Trong toán sau, câu a) anh Phạm Kim Hùng, câu b) kết mạnh mà chúng tơi tìm

Bài tốn 3. Choa, b, c, d≥0, a+b+c+d = ĐặtFk = (1 +ak)(1 +bk)(1 +

ck)(1 +dk) Chứng minh rằng: a) F4F3

b) F2F1

Lời giải:

a) Ta chứng minh BĐT phản chứng Giả sử ngược lại tức tồn bốn số (a, b, c, d) thỏa mãn: a, b, c, d ≥ 0, a+b+c+d =

F4F3 (1)

Theo BĐT Bunhacôpski ta có: F4.F2F32 , F3.F1F

2 , F2.F0F2

1 (2) Từ (1) (2) suy F4 < F3< F2 < F1 < F0 = 16 (3) Từ (3) ta

coù F4 <16 suy max(a, b, c, d)<2

Để dẫn tới mâu thuẫn với (3), ta chứng minh F3F1 (4) Thật

vaäy:

(4) ⇔(1−a+a2)(1−b+b2)(1−c+c2)(1−d+d2)≥1

⇔(3 4+

(2a−1)2

4 )(

3 4+

(2b−1)2

4 )(

3 +

(2c−1)2

4 )(

3 4+

(2d−1)2

4 )≥1

⇔(1 + (2a−1)

2

3 )(1 +

(2b−1)2

3 )(1 +

(2c−1)2

3 )(1 +

(2d−1)2

3 )≥

4

4

⇔(1 +x2)(1 +y2)(1 +z2)(1 +t2)≥

"

1 +

x+y+z+t

4

2#4

(5) (Trong x= 2a√−1

3 , y= 2√b−1

3 , z = 2√c−1

3 , t = 2√d−1

3 )

Từ xét BĐT

(1 +A2)(1 +B2)≥

"

1 +

A+B

2

2#2

(6)

8(AB)

2

(8−A2−6ABB2)≥0 Ta thấy A+B ≤2 BĐT

Khơng tổng quát giả sử abcd Kết hợp với a+b+

(28)

BÑT(6) ta coù:

(1 +x2)(1 +t2)≥

"

1 +

x+t

2

2#2

(7)

(1 +y2)(1 +z2)≥

"

1 +

y+z

2

2#2

(8) nhaân (7) (8) vế theo vế suy ra:

(1 +x2)(1 +t2)(1 +y2)(1 +z2)≥

1 + (x+t )

2

1 + (y+z )

2

2

(9) Từ x +t < y+z < suy ra: x+2t + y+2z < Do lại áp dụng

BĐT(6) ta được:

1 + (x+t )

2

1 + (y+z )

2

"

1 +

x+y+z+t

4

2#2

(10) Từ (9) (10) suy ra:

(1 +x2)(1 +y2)(1 +z2)(1 +t2)≥

"

1 +

x+y+z+t

4

2#4

Vậy (5) suy (4) (mâu thuẫn với (3)) Điều có nghĩa việc giả sử (1) sai tức ta có BĐT ngược lại F4F3 (đpcm)

b) Câu mạnh câu a) dùng "mánh lới" câu a khơng ổn, nhiên " đường lớn tiến cơng" khơng gặp vấn đề gì:

Đặt f(a, b, c, d) = V TV P ta cần chứng minh f(a, b, c, d) ≥ Muốn

vậy, trước hết ta chứng minh mệnh đề sau: Mệnh đề: Nếu a+b≤2 vàaxb f(a, b, c, d)−f(x, a+bx, c, d)≥0 Thật vậy:

f(a, b, c, d)−f(x, a+bx, c, d)

= (ax)(xb) [(d+ 1)(c+ 1)−(d2+ 1)(c2+ 1)(abx2+ax+bx−2)]

(29)

Trở lại toán ta giả sử abcd Đặtx = a+3b+c thì: Chú ý a+c≤2 vàcxa nên áp dụng mệnh đề ta có:

f(a, b, c, d)≥f(a+cx, b, x, d) (1)

Chú ý x = (a+cx3)+b+x nên x = min{x, b, a+ cx} x =

max{x, b, a+cx}thìa+cx=b=xnênf(a+cx, b, x, d) =f(x, x, x, d) tốn cịn biến

Giả sử ngược lại, có hai trường hợp:

b < x < a+cx (2) a+cx < x < b (3) Lại sử dụng mệnh đề cho ta:

(2) :f(a+cx, b, x, d)≥f(x, a+b+c−2x, x, d) =f(x, x, x, d) (3) :f(a+cx, b, x, d)≥f(a+b+c−2x, x, x, d) =f(x, x, x, d) Nói chung trường hợp ta có

f(x, b, a+cx, d)≥f(x, x, x, d) (2)

Từ (1) (2) suy f(a, b, c, d)≥f(x, x, x, d) Để giải toán biến,

ta thay x= x+y3+z = 4−3d chứng minh:

f(4−d ,

4−d

3 , 4−d

3 , d)≥0 (4) Thaät vaäy:

(4) ⇔

729(d

6−

22d5+ 223d4−1268d3+ 4210d2 −7564d+ 6364)(d−1)2 ≥0 Bất đẳng thức cuối nên ta có điều phải chứng minh

*Nhận xét: Việc đổi biến trước dồn biến câu a) kì lạ đem lại hiệu không ngờ Kĩ thuật dồn biến câu b) mạnh, hoàn toàn sơ cấp (bởi số bước dồn biến hữu hạn) Kĩ thuật ứng dụng cực tốt cho biến Hơn thế, phần sau mở rộng để giải toán với n biến

Cuối cùng, đến với ví dụ cho trường hợp dồn biến biên Đây toán anh Phạm Kim Hùng

Bài toán 4. Cho a, b, c, d≥0 Chứng minh rằng:

a

b2 +c2+d2 +

b

a2+c2+d2 +

c

a2+b2+d2 +

d

a2+b2 +c2 ≥

4

(30)

Lời giải: Xét

f(a, b, c, d) = X

4

a

b2+c2+d2 −

4

a+b+c+d

Giả sử abcd Ta có: f(a, b, c, d)−f(a, b,

a2+b2,0)

= c

a2+b2 +d2 +

d

a2+b2+c2 −

4

a+b+c+d −(

c2+d2 a2+b2 −

4

a+b+

a2+b2)

(do a2+b2 ≥c2+d2 nên dễ thấy BĐT đúng)

Vậy vấn đề lại chứng minh f(a, b,

c2+d2,0)≥0 BĐT cuối chứng

minh khơng khó nên xin nhường lại cho bạn đọc

*Nhận xét: Cách dồn biến nhằm bảo toàn tổng a2+b2+c2+d2.

Tất nhiên, việc điều quan trọng, thích bạn bảo toàn a+b+c+d cách chứng minh f(a, b, c, d)≥ f(a+2b + a+2b, c, d), sau đánh giá

f(t, t, c, d)≥ 2t

t2+ (c+d)2 +

c+d

(c+2d)2+ 2t2 −

4 2t+c+d

= x

x2/4 +y2 +

y

y2/4 +x2/2 −

4

x+y (trong x= 2t, y =c+d)

Bước cuối f(x, y)≥0 (chứng minh khơng khó bạn giả sử x+y= cho gọn)

Đến tạm kết thúc phần dồn biến cho BĐT "cụ thể" (có biến) để bước sang phần dồn biến cho BĐT n biến Như chúng

(31)

6 Dồn biến hàm lồi.

Các bạn thân mến, phương pháp dồn biến mà tìm hiểu mục trước từ trời rơi xuống Thật ý tưởng dồn biến thể rõ BĐT cổ điển Do xếp theo dịng chảy thời gian lẽ mục phải nêu từ đầu Tuy nhiên, nghĩ thú vị trở lại gốc rễ sau bạn cảm nhận dồn biến phương pháp "hiện đại"

Một công cụ để dồn biến BĐT "dạng cổ điển" hàm lồi Đây khái niệm quen thuộc, nhiên để tiện lợi cho bạn đọc xin nhắc lại

Định nghĩa: Một hàm sốf : [a, b]→R gọi lồi nếu:

f(tx+ (1−ty))≤tf(x) + (1−t)f(y),x, y∈[a, b],t∈[0,1]

*Nhận xét:

1) Nếu f khả vi lần tiêu chuẩn quan trọng để kiểm tra tính lồi

f00(x)≥0,x∈(a, b)

2) Nếu f lồi f liên tục Ngược lại, f liên tục tính lồi f

tương đương với điều "yếu hơn" là: f(x+2y)≤ f(x)+f(y)

2

Các bạn thấy, định nghĩa hàm lồi đánh vào mục tiêu dồn biến Chúng ta có kết quen thuộc sau:

Định lý: (BĐT Jensen) Cho f hàm số lồi [a, b]→R

(i) Với xi n số thuộc [a, b]ta có:

f(x1+x2 + +xn

n )≤

f(x1) +f(x2) + +f(xn)

n

(ii) Với xi n số thuộc A λi n số khơng âm có tổng ta có:

(32)

Bài tốn 1. (BĐT Cauchy) Cho n số thực dươngxi Chứng minh rằng: x1+x2+ +xn

n

n

x1x2 xn Lời giải:

Lấy logarit vế, ta chuyển dạng:

ln(x1+x2+ +xn

n )≥

ln(x1) +ln(x2) + +ln(xn)

n

Hàm số f(x) =ln(x)đi từR+ →R khả vi lần vàf00(x) =−x−2 <0,x >

0 Do hàm g(x) = −f(x) thỏa g00(x)> 0,x >0 Vậy g lồi Từ đó,

áp dụng BĐT Jensen ta có điều phải chứng minh

*Nhận xét: Một cách khác thông dụng dùng để chứng minh BĐT Cauchy, chứng minh quy nạp theo n Cách làm hay, ta có cảm

giác "cái cho n = cho n tùy ý" Các bạn

quan sát kĩ cách chứng minh đó, chứng minh lại BĐT Jensen, bạn thấy hàm lồi tổng quát nói lên chất vấn đề

Hàm lồi ứng dụng nhiều BĐT cổ điển, BĐT cổ diển lại giải nhiều tốn khác Tất nhiên, khơng phải công cụ "vạn năng", nhiên biết sử dụng khéo léo sức mạnh khơng nhỏ Chúng tơi dẫn ví dụ cho thấy áp dụng hàm lồi kết quả, song giúp giải trường hợp quan trọng mà trường hợp lại chứng minh đơn giản cách hay cách khác

Bài toán 2. Cho số thực x, y, z có tổng Chứng minh rằng: x

1 +x2 + y

1 +y2 + z

1 +z2 ≤

9 10 Lời giải:

Xét f(t) = 1+tt2 BĐT cần chứng minh tương đương:

f(x) +f(y) +f(z)≤3f(x+y+z

(33)

Do đó, −f hàm lồi coi tốn giải

Ta có:

f00(t) = 2t(3−t

2

) (1 +t2)3

nên −f00(t)≥0,t ∈[0,√3] Vậy nếux, y, z∈[0,√3]thì tốn giải

quyết

Trong trường hợp cịn lại chắn ta có dấu BĐT thực Do việc chia thành nhiều trường hợp để xét

Có thể giả sử xyz lưu ý x+y+z = vàx, y, z /∈[0,1] nên z phải

âm suy f(z)<0

*Nếu y âm suy x dương vàf(y)< 0, ta có f(x) +f(y) +f(z) < f(x)<

1/2<9/10

*Nếu ydương suy rax dương lưu ýf(y), f(x)nghịch biến trên[

3,+∞] f(x) +f(y) +f(z)< f(x) +f(y)< f(

3) +f(

3)<9/10 Bài toán chứng minh xong

*Nhận xét: Tất nhiên lời giải chưa phải ngắn gọn so với nhiều lời giải khác cho tốn mà chúng tơi biết Tuy nhiên tư tưởng hồn tồn sáng Ở đây, thay mong muốn dồn biến tồn cục (dồn lần biến) việc hi vọng hợp lý dồn biến lời giải ngắn Thật vậy, có biến x, y, z

thuộc đoạn [0,

3]thì dùng hàm lồi ta dồn biến nhau, toán biến, xem giải xong Trong phần cịn lại việc chia trường hợp đơn giản Như vậy, có thêm kĩ thuật để dồn biến sử dụng hàm lồi

Mặc dù công cụ tốt, điểm yếu dễ nhận BĐT, biến phải nằm biểu thức độc lập (để viết thành dạng f(x1) + +f(xn)) Trong đó, BĐT mà ta gặp

phần lớn khơng có điều đó, ta phải làm việc với dạng tổng quát f(x1, , xn) Chúng ta phải thiết lập kết dồn biến cho dạng

tổng quát mục sau

(34)

Định lý: Cho f : [a, b]→R hàm lồi Khi đó: f(x)≤max{f(a), f(b)},x∈[a, b] Chứng minh:

f liên tục nên f đạt giá trị lớn x0 ∈[a, b] Xét |x0a| ≤ |x0b| (nghĩa x0 gần a b) Thì x1 = 2x0a ∈ [a, b] Khi theo

định nghóa hàm lồi ta có:

f(a) +f(x1)≥2f(a+x1

2 ) = 2f(x0)

suy f(a) =f(x0) Với x0 gần b a chứng minh tương tự

*Nhận xét: Để bạn cảm nhận "cái đúng" định lý chúng tơi nêu hình ảnh f00(x)> 0,x ∈ (a, b) Khi đó, f0 đồng biến

nên có tối đa nghiệm (a, b), nói cách khác đổi dấu tối đa

lần Do f rơi vào trường hợp sau đây: đồng biến, nghịch biến, "đi

lên xuống", "đi xuống lên" Và trường hợp ta thu kết cần thiết (Một chứng minh khác trường hợp giả sử f đạt cực đại x0 ∈(a, b)thì f00(x0)≤0, mâu thuẫn.)

Chúng tơi dẫn toán mà chúng thực tốn khó cho dù giải biến đổi đại số hay quy nạp

Bài toán 3. Cho 0< p < q, n số thực xi∈[p, q] Chứng minh rằng: (x1+x2+ +xn)(1

x1 +

1

x2 + +

1

xn

)≤n2 +

n2

4

(pq)2 pq

trong kí hiệu [x]là phần nghuyên x

(*Ghi chú: Đây tổng quát, trường hợp n = USAMO 77, n= đề thi Olympic 30−4 năm 2001 )

Lời giải:

Từ giả thiết xi ∈ [p, q], ta dễ dàng đoán rằng: GTLN đạt

xi ∈[p.q]với i Khi đó, g/s n số xik số pnk số q thì:

V T = (kp+ (nk)q)(k

q + nk

q ) =k

+ (nk)2+k(nk)(p

(35)

=n2+k(nk)(pq)

2

pq =n

2

+1

n2−(n−2k)2(pq)

2 pq

Vì k nguyên nênn2−(n−2k)2 ≤n2(khi n chẳn) vàn2−(n−2k)2 ≤n2−1

(khi n lẻ) Từ đó, ta thu BĐT ban đầu đồng thời trường hợp dấu xảy

Đến đây, ta nhận ra: mấu chốt vấn đề nhận xét: "GTLN đạt xi = p xi =q với i" Và thật bất ngờ, nhận xét chứng minh dễ

Với i, ta xem vế trái hàm theo xi, ta chứng tỏ: f(xi)max{f(p), f(q)}, dấu xảy xi ∈ {p, q}

Ta có: f(x) =Ax+Bx +C Có thể khảo sát hàm để kết (suy

ra dấu xảy xi ∈ {p, q}) Song trình bày cách sơ cấp Để ý:

f(xi)f(p) = (xip)(A

B xip

)

f(xi)f(q) = (xiq)(A

B xiq

)

Từ f(xi)> max{f(p), f(q)} rõ ràng xi ∈ {/ p, q} và:

AB

xip

>0, AB xiq

B

xip

< A < B xiq mâu thuẫn p < q Vậy f(xi)max{f(p), f(q)}

Cần nói thêm trường hợp dấu bằng: g/s f(xi) =max{f(p), f(q)}mà xi ∈ {/ p, q} Nếu f(xi) = f(p) A = xB

ip >

B

xip, f(xi)−f(p) <

(mâu thuẫn) Tương tự, f(xi) = f(q) mâu thuẫn Vậy f(xi) =

max{f(p), f(q)}tương đương với xi ∈ {p, q} *Nhận xét: Ta có tốn mở rộng sau:

"Cho ai ∈[a, A], bi∈ [b, B] với 0< aA 0< bB Tìm giá trị lớn

T = (a

2

1+ +a

n)(b

2

1+ +b

n)

a1b1+ +anbn

.” Nhà toán học Polya cho chặn là:

4

q

AB ab +

q

ab AB

2

(36)

một điều tự nhiên đặt chặn BĐT Côsi ? Nếu bạn tị mị xem tiếp toán sau đây:

Bài toán 4. (Phan Thành Nam) Cho < a < b, n số thực xi ∈ [p, q] Chứng minh rằng:

T = x1

x2 + x2

x3 + + xn

x1n+

hn

2

i(pq)2 pq

Lời giải:

Với i, thay xi p hay q trường hợp T phải tăng lên, T không tăng buột xi ∈ {p, q}.

Cho i chạy từ tới n, với i ta thay xi p hay q cho T tăng lên (hoặc giữ nguyên hai trường hợp không tăng)

Sau bước biến đổi ta có xi ∈[p, q]với i.Nếu xi=q với

i T = n, khơng phải GTLN, cần xét ∃xi =p.Do hốn vị vịng quanh nên giả sử x1 =p. Khi x3 =p hay q ta thay x2 q T không giảm Sau thay x2 q ta lại thay x3 p

thì T khơng giảm Cứ ta xen kẽ p, q số xn T khơng giảm Sau thực q trình lúc ta có

T = n 2(

p q +

q

p) (nếu n chẵn) T = n−1

2 (

p q +

q

p) + (nếu n lẻ)

Ta viết lại trường hợp dạng:

T =n+hn

i(pq)2

pq ,n

và vế phải BĐT cần chứng minh Đẳng thức xảy

xi ∈ {p, q} xen kẽ kể từ x1 tới xn (khơng kể vịng xn, x1) Bài tốn đến giải trọn vẹn !

(37)

biệt trường hợp cực trị đạt tâm, hàm lồi cho ta kiểu dồn biến thú vị mà tìm hiểu mục sau Mặc dù với loạt BĐT xuất gần cơng cụ cổ điển khơng đủ (hoặc khó khăn), lần nữa, nhấn mạnh tầm quan trọng ý tưởng "cổ điển", mà dựa vào "đứng vai người khổng lồ"

7 Dồn biến giá trị trung bình.

Cho đến bây giờ, phương pháp dồn biến chúng ta, số lần thực thao tác dồn biến ln hữu hạn, nhờ lời giải rõ ràng hoàn toàn sơ cấp Đây điều tốt mà muốn trì tiếp tục mục

Trước hết, giới thiệu thêm cách dồn biến dành cho hàm lồi Ta gọi kĩ thuật dồn biến giá trị trung bình, mà bạn thấy rõ điều qua kết sau:

Định lý: Cho f hàm lồi [a, b]→R Ta coù:

f(a) +f(b)≥f(x) +f(a+bx),x∈[a, b] Chứng minh:

x ∈ [a, b] nên: x = ta+ (1−t)b với t ∈ [0,1] Khi đó: a+bx = (1−t)a+tb Áp dụng định nghĩa hàm lồi, ta có:

f(x) +f(a+bx) =f(ta+ (1−t)b) +f((1−t)a+tb)

≤[tf(a) + (1−t)f(b)] + [(1−t)f(a) +tf(b)] = f(a) +f(b)

Ứng dụng kết này, ta có chứng minh cho BĐT Jensen Nhắc lại: Định lý: (BĐT Jensen) Cho f hàm số lồi [a, b]→ R Thì với xi ∈ [a, b] n số có trung bình cộng T, ta có:

f(x1) +f(x2) + +f(xn)nf(T) Chứng minh:

Ta cho thực thuật toán sau:

(38)

*Bước 2: Vì khơng có xi = T,i nên phải có biến lớn hơn T biến nhỏ T, mà ta giả sử x1 > T > x2 Khi thay

(x1, x2, , xn) (T, x1+x2T, , xn) Sau trở lại bước 1.

Như lần thực bước có trung bình cộng T, nhiên làm cho biểu thức f tăng lên Mặt khác lần thực

hiện bước số biến T tăng lên 1, sau hữu hạn (có thể lấy n−1) lần thực bước 2, ta phải dừng lại bước Chú ý

là trình thay biểu thức f tăng lên, ta có điều phải

chứng minh

Vậy có thêm cách dồn biến Sỡ dĩ không dưa cách dồn biến mục trước, có giá trị dồn biến tâm, mà với n = kĩ thuật dồn biến đủ sử dụng Tuy nhiên, kĩ thuật phát huy tác dụng số biến tăng lên, cụ thể với trường hợp n biến tổng quát Lý đơn giản: BĐT với n biến, cho dù ta dồn biến chưa thu đáng kể, trường hợp sau hữu hạn lần dồn biến đưa trường hợp biến (chứ chưa nói đưa trường hợp biến nhau) Tuy nhiên, sử dụng kĩ thuật dồn biến biên dồn biến giá trị trung bình tình hình lại khác: sau lần dồn biến số lượng biến có giá trị cố định tăng lên (là giá trị biên giá trị trung bình), cần hữu hạn lần dồn biến ta đưa tất biến giá trị cố định toán xem giải xong

Tất nhiên, khả để dồn biến biên giá trị trung bình khơng cao Tuy nhiên, quan trọng tinh thần nó: dồn biến giá trị cố định Bạn đọc thấy ý tưởng hiệu trường hợp biến (xem câu c), Bài tốn 3, $5) Trong mục này,

chúng tơi tiếp tục giới thiệu tốn khác, mà ý tưởng dồn biến giá trị trung bình cho lời giải bất ngờ Đây toán đặc sắc anh Phạm Kim Hùng, mà việc giải chúng đem lại cho nhiều ý tưởng cho phương pháp dồn biến

Bài toán 1. Cho n số thực dương a1, a2, , an có tích Chứng minh với k= 4(n−1) ta ln có:

1

a1 +

1

a2 + +

1

an

+ k

a1+a1+ +an

n+k

n (1)

(39)

Với n = 1, n = tốn đơn giản, nên ta xét n

Trước hết, ta khảo sát trường hợp dồn biến rút ra:

Mệnh đề 1: Kí hiệu f(a1, a2, , an) biểu thức vế trái BĐT cần chứng minh

(i) Nếu a1xa2a1a2 ≤1thì

f(a1, a2, , an)f(x,a1a2

x , a3, , an)

(ii) Nếu (1−a1)(1−a2)[ka1a2−(Pin=1ai)(Pni=3ai+a1a2 + 1)]≥0thì

f(a1, a2, , an)f(1, a1a2, a3, , an) (iii)Nếu a1, a2 ≥1≥a3 thì:

f(a1, a2, , an)min{f(1, a1a2, a3, , an), f(1, a2, a1a3, a1ai, , an)} Chứng minh mệnh đề 1:

Để viết cho gọn ta đặt A=Pni=3ai.

(i) Ta coù:

f(a1, a2, , an)f(x,a1a2

x , a3, , an)

=

a1 +

1

a2

1

xx a1a2 +

k

A+a1+a2

k A+x+ a1a2

x

= (xa1)(a2x)[(A+a1+a2)(A+x+ a1a2

x )−ka1a2]

a1a2(A+a1 +a2)(A+x+ a1a2

x ) Theo BÑT Cauchy:

(A+a1+a2)(A+x+a1a2

x ))≥n

≥4(n−1) =kka1a2

vaø ta có đpcm

(ii) Cũng từ đẳng thức cho x= ta có:

f(a1, a2, , an)f(1, a1a2, a3, , an)

= (1−a1)(1−a2)[ka1a2−(A+a1+a2)(A+a1a2+ 1)]

a1a2(A+a1+a2)(A+a1a2+ ta có đpcm

(40)

Trường hợp 1: Nếu ka1a2 ≥(Pni=1ai)((Pni=3ai+a1a2+ 1)])thì dùng (ii)ta có f(a1, a2, , an)f(1, a1a2, a3, , an).

Trường hợp 2: Nếuka1a2 ≤(Pni=1ai)((Pni=3ai+a1a2+1)])thì vìa3 ≤1≤a2

nên:

ka1a3≤ ( n

X

i=1

ai)(X i6=1,3

ai+a1a3+ 1) (thaät vaäy:

P

i6=1,3ai+a1a3+

a1a3 =

Pn

i=1aia1a3+

a1a3 +

Pn

i=1aia1a2+

a1a2 + =

Pn

i=3ai+a1a2+

a1a2 )

Do đó, dùng (ii) ta có: f(a1, a2, , ai, , an)f(1, a2, , a1ai, , an). Mệnh đề chứng minh xong! Nó cho phép ta đưa toán biến Mệnh đề 2: Ta ln đưa tốn trường hợp có n −1 biến ≤1

Chứng minh mệnh đề 2:

*Bước 1: Đưa trường hợp có n−1 biến ≤1

Giả sử cịn có nhiều biến lớn 1, mà ta giả sử a1, a2

Thì sử dụng mệnh đề 1(iii)ta ln thay bộ(a1, , an)bởi khác, có tích 1, làm cho f khơng tăng, có số biến

tăng lên Do sau hữu hạn lần thay (không n−1) ta

n−1 biến≤1

*Bước 2: Đưan−1biến ≤1

Giả sử a1a2 an−1 ≤ n −1 biến có trung bình nhân x Nếu n−1 biến chưa a1 < x < an−1 dùng mệnh

đề (i) ta thay (a1, a2, , an−1, an) (x, a2, ,

a1an−1

x , an) Khi f khơn giảm số biến x tăng lên Ta lưu ý

a1an−1

xa1

x ≤1(vìa1 số nhỏ trongn−1sốa1, , an−1 nên a1x), việc thay đảm bảon−1biến ≤1, điều cho phép

việc thay thực liên tiếp Vậy sau hữu hạn (không quán−1)

lần thay ta có n−1biến ≤1

Cuối cùng, ta giải toán biến, tức chứng minh:

f(x, x, , x,

(41)

Đặt:

g(x) := f(x, x, , x, xn−1) =

n−1

x +x

n−1

+ k

(n−1)x+ xn1−1

với x∈(0,1]

Ta coù:

g0(x) =−n−1

x2 + (n−1)x

n−2

k[n−1−

n−1

xn ]

((n−1)x+xn1−1)2

= (n−1)x n−1

x2

(n−1)xn−1 (n−1)xn+ 1

2

lưu ý k = 4(n−1) Ta thấy g(x)≤0với x∈(0,1], nên g(x)≥ g(1) ta có đpcm Bài tốn chứng minh xong!

*Ghi chú: Bài toán ban đầu anh Phạm Kim Hùng với k = 3n, n

Kết mạnh hơn, bạn thấy chứng minh cho trường hợp biến só k = 4(n−1) "hợp lý"

Bài tốn 2. Cho n số thực dương a1, a2, , an có tích Chứng minh rằng:

(1 +a21)(1 +a

2) (1 +a

n)≤ 2n

n2n−2(a1+a2+ +an) 2n−2

Lời giải:

Với n = 1, n = đơn giản nên ta chứng minh cho n ≥ Ta

thấy toán tương đương với f(a1, a2, , an)≥0 tương đưong với

g(a1, a2, , an)≥0, đó:

f(a1, a2, , an) =k(a1+a2+ +an)2n−2−(1 +a21)(1 +a22) (1 +a2n)

g(a1, a2, , an) = ln(k) + (2n−2) ln(a1+a2+ +an)+

−ln(1 +a21)−ln(1 +a

2)− −ln(1 +a

n) (về việc phải xét fg bình luận sau)

Khảo sát sơ trường hợp dồn biến, ta có:

Mệnh đề 1:

(i) Nếu a1 ≥1≥a2, a3 thì:

(42)

(ii) Nếu a1 =max{ai}ni=1 a1xa2 ≥1 thì: g(a1, a2, , an)g(x,a1a2

x , a3, , an)

Chứng minh mệnh đề 1: (i) Xét hiệu

f(a1, a2, , an)f(1, a1a2, a3, , an) =ks2n−2−ku2n−2+ [2(1 +a21a

2

2)−(1 +a

1)(1 +a

2)](1 +a

3) (1 +a

n) (vớis=a1+a2 + +an, u= +a1a2+ +an) =k(a1+a2−1−a1a2)(s2n−3+s2n−4u+ +u2n−3)+

+(1−a21)(1−a

2)(1 +a

3) (1 +a

n) =−(1−a1)(1−a2)[k(s2n−3+ +u2n−3)+

−(1 +a1)(1 +a2)(1 +a23) (1 +a2n)] Sử dụng lại đẳng thức với a3 đổi chỗ cho a1, ta có:

f(a1, a2, , an)f(a1,1, a2a3, , an) =−(1−a2)(1−a3)[k(s2n−3 + +v2n−3)+

−(1 +a2)(1 +a3)(1 +a21)(1 +a

4) (1 +a

n)] (với v= +a2a3+a1+a4+ +an) Từ đẳng thức trên, ta thấy:

∗ neáu k(s2n−3+ +u2n−3)−(1 +a1)(1 +a2)(1 +a23) (1 +a

n)≥0 (2) Thì f(a1, a2, , an)f(1, a1a2, a3, , an)

∗neáuk(s2n−3+ +v2n−3)−(1+a2)(1+a3)(1+a2 1)(1+a

2

4) (1+a

n)≤0 (3) Thì f(a1, a2, , an)f(1, a1a2, a3, , an)

Do đó, ta cần chứng minh BĐT (2) (3) có xong! Chẳng hạn, ta giả sử (2) sai, chứng minh (3) Muốn vậy, ta cần chứng minh: uvvà(1+a1)(1+a23)≤(1+a3)(1+a

2 1)

là xong! Điều có từ việc tính tốn đơn giản:

uv=a3+a1a2a1a2a3 = (1−a2)(a1a3)≥0 (1 +a1)(1 +a23)−(1 +a3)(1 +a

2

1) = (a3a1)(a1a3 +a1+a3−1) ≤0

Vậy mệnh đề (i) chứng minh xong!

(43)

vào ta dùng đạo hàm Xét:

g(t) = ln(k) + 2(n−1) ln(ta1+a2

t +a3 + +an)+ −ln(1 +t2a21)−ln(1 +

a2

t2)−ln(1 +a

3)− −ln(1 +a

n) với t ∈[pa2/a1,1]

Ta coù:

g0(t) = 2(n−1)(a1a2

t2)

ta1+ a2

t +a3+ +an

− 2ta

2 1−

2a2

t3

(1 +t2a2 1)(1 +

a2

t2)

= 2(a1a2 t2)[

(n−1)

ta1+ a2

t +a3+ +an

ta1+

a2

t (1 +t2a2

1)(1 +

a2

t2)

]

t ∈[pa2/a1,1] neân a1a2

t ≥ Do đó, gọi T thừa số cịn lại, ta cần chứng minh T ≥0 suy rag đồng biến (trên [pa2/a1,1])

Để viết cho gọn, ta đặt

c=

r

(1 +t2a2 1)(1 +

a2

t2), d=ta1+ a2

t

Ta coù:

T ≥0⇔ n−1

d+a3+ +an

d

c2 ⇔(n−1)c

d2+d(a3+ +an)cd (BĐT Bunhiacopski) nên để có BĐT ta cần:

(n−2)ca3+ +an Điều c > a1a2a1max{a3, , an}

Laáy t0 =max{x,a1a2

x }/a1,

t0 ∈[

r

a2

a1,1], t0a1=max{x, a1a2

x }, a2

t0 =min{x, a1a2

x }

(44)

Trở lại tốn, ta nói (a1, a2, , an) thay (b1, b2, , bn)) f(a1, a2, , an)f(b1, b2, , bn) hoặc g(a1, a2, , an)g(b1, b2, , bn).

Mệnh đề 2: Luôn đưa trường hợp có n−1 biến ≥1

Chứng minh mệnh đề 2:

*Bước 1: Đưa trường hợp có n−1 biến ≥1

Giả sử cịn có biến a2, a3<1 Khi phải có biến>1, mà ta

giả sử a1 Sử dụng mệnh đề (i), ta thay (a1, a2, , an) (1, a1a2, a3, , an) (a1,1, a2a3, , an) Chú ý cho dù thay bộ

nào, số biến tăng lên Do đó, động tác thay phải dừng lại sau khơng q n−1 lần Khi đó, ta có n−1 biến ≥1

*Bước 2: Ta chứng minh ln thay n−1 biến ≥ trung bình nhân chúng Thật vậy, giả sử a1a2 an−1 ≥ ≥ an đặt x = n−√1 a1, a2, , a

n−1 ≥ Nếu n−1 biến cịn

biến khác x a1 > x > an−1 Sử dụng mệnh đề (ii) ta thay

(a1, a2, , an−1, an) (x, a3, ,a1xa2, an) Chú ý là

a1an−1

xan−1 ≥ (vì a1 số lớn số {ai}in=1−1 nên a1x) việc thay

thế đảm bảo n −1 biến ≥ (để thay liên tiếp) Chú ý sau thay số biến xtăng lên

Do đó, sau khơng qn−1lần thay cản−1biến bằngx

Cuối ta giải toán biến Xét hàm số h(x) :=g(x, x, , x,xn1−1)

= ln(k) + 2(n−1) ln((n−1)x+

xn−1)−(n−1) ln(1 +x

)−ln(1 +

x2n−2)

với x≥1

Ta coù:

h0(x) = 2(n−1) n−1− n−1

xn

(n−1)x+xn1−1

−2(n−1)x

1 +x2 −

−2(n−1)

x2n−1

1 + x2n1−2

= 2(n−1)

x

(n−1)(xn−1) (n−1)xn+ 1 −

x2

1 +x2 +

1 +x2n−2

= 2(n−1)

x

(n−1)(xn−1) (n−1)xn+ 1 −

x2n−1 (1 +x2)(1 +x2n−2)

(45)

Chú ý x≥1nên để có h0(x)≥0 ta cần:

n−1

(n−1)xn+ 1 ≥

xn+ (1 +x2)(1 +x2n−2)

Ta đạt điều đánh giá đơn giản:

n−1

(n−1)xn+ 1 ≥

xn+ 1 ≥

xn+ (1 +x2)(1 +x2n−2)

(Có dấu ≥ thứ hai BĐT Bunhiacopski )

Vậy với x≥1 h0(x)≥0 nên h(x) đồng biến, suy h(x)≥h(1) = 0

ta có đpcm

Vậy tốn chứng minh xong! Đẳng thức xảy a1 =a2=

=an= với n≥3 (còn với n= 1, n= có đẳng thức) *Nhận xét:

1) Bài toán anh Phạm Kim Hùng đặt dạng toán mở chứng minh chứng minh cho

2) Ở việc xét đồng thời hàm f, g cho phép ta mở rộng khả dồn

biến: xét f đơn giản hơn, xét g đơn giản Trong tốn

1 vấn đề đơn giản nên cần hàm f đủ

8 Định lý dồn biến tổng quát.

Các bạn thân mến, nói định lý dồn biến phải nhắc tới kết ấn tượng, định lý dồn biến mạnh (SMV) anh Phạm Kim Hùng định lý dồn biến không xác định (UMV) bạn Đinh Ngọc An Trong đó, "xương sống" định lý bổ đề dãy số, kết cho ta cảm giác rõ ràng dồn biến

Trong mục này, cung cấp cho bạn định lý dồn biến tổng quát − định lý GMV anh Phan Thành Nam − với

cách tiếp cận Có thể trình bày ngắn gọn cách dẫn định lý chứng minh nó, nhiên chúng tơi khơng làm muốn chia với bạn đường (trong tư duy) để xây dựng Hi vọng sau xem xong, bạn có cảm giác có vơ số định lý dồn biến

(46)

Định nghóa 1:

• Khơng gian Rn tập hợp thứ tựx= (x1, x2, , xn) vớix

iR,i

•Một dãy {xm= (x1,m, , xn,m)} trongRn gọi hội tụ z = (z1, , zn)

Rn dãy xi,m hội tụ zi m→ ∞, ∀i= 1,2, , n

• Cho DRn Một hàm số f :DR gọi liên tục D nếu: với dãy {xm} ⊂ D với zD cho {xm} hội tụ z, ta có:

f(xm) hội tụvề f(z)

Định nghóa 2: Cho DRn Ta nói:

• D đóng với dãy {xm} ⊂ D với zRn cho {xm} hội tụ z, ta có zD

• D bị chặn tồn số thực M cho: ∀x = (x1, , xn)D}, |xi| ≤M,i= 1,2, , n

Ví dụ tập hợp hữu hạn đóng bị chặn Xuất phát điểm kết tuyệt đẹp sau đây:

Định lý 1: Cho D đóng bị chặn Rn, f : DR liên tục Thì f đạt giá trị nhỏ D, nghĩa tồn x0D cho: f(x0)≤f(x),xR

Đây kết có chương trình phổ thơng nước, nhiên nước ta xem thuộc "Toán cao cấp" Tuy nhiên, để tiện lợi cho bạn đọc dẫn chứng minh mà bạn hồn tồn hiểu với kiến thức phổ thông

Chúng nhắc lại kết có SGK: " dãy số thực đơn điệu bị chặn hội tụ" "Tiên đề" sử dụng để chứng minh kết dãy

Định nghĩa 2: Cho dãy số {am}∞m=1 (trong R R

n) Moät daõy

{amk}

k=1 gọi dãy dãy {am}∞m=1 nếu{mk}∞k=1 dãy

tăng ngặt số nguyên dương

*Ví dụ: {a2m}∞m=1 dãy dãy {am}∞m=1 Dưới cận

của số bỏ qua không gây hiểu lầm

Bổ đề 1: (Weierstrass) Mỗi dãy am bị chặn R có dãy hội tụ Chứng minh:

Ta chứng minh có dãy đơn điệu xong Xét tậpT :={mZ+|∃m0 > m cho a

(47)

dãy {am} giảm kể từ số Nếu T vơ hạn ta trích dãy tăng Trong hai trường hợp ta ln có dãy đơn điệu Bổ đề 2: (Weierstrass) Mỗi dãy am bị chặn trongRn có dãy hội tụ Chứng minh:

Xét {am = (x1,m, , xn,m)} dãy bị chặn Rn Khi dãy

{x1,m} bị chặn R nên có dãy {x1,mk1} hội tụ Dãy {x2,mk1}

cũng bị chặn R nên có dãy {x2,mk

2} hội tụ Bằng cách lấy

"dãy dãy con" liên tiếp vậy, cuối ta thu dãy

{amk = (x1,mk, , xn,mk}mà∀i= 1,2, , n, ta có dãy{xi,mk}hội tụ R Điều có nghĩa dãy {amk} hội tụ R

n

Bổ đề 3: (Tính đầy đủ R) Cho A tập bị chặn R Thì tồn

MR cho: MA (nghóa Ma,aA) có dãy{ak} A hội tụ M Ta kí hieäu M =inf A

Chứng minh:

Ta chứng minh ∀ε > 0, ∃aA, aεA Giả sử ngược lại Khi

đó lấy x1A tùy ý, quy nạp ta xây dựng dãy {xm} A cho xm+1 ≤xmε, ∀mZ+ Khi ta có: xmx1−(m−1)ε, ∀mZ+ điều mâu thuẫn với A bị chận

Như vậy, ∀mZ+, tồn a

mA choamm1 ≤A Vì dãy{am}bị chặn nên có dãy {amk} hội tụ M R Ta chứng minh MA

nữa xong Thật vậy, lấy aA amk −

1

mk ≤ a, ∀kZ

+, neân

cho k→ ∞ suy Ma

Chứng minh định lý 1: Xét A = f(D) Ta chứng minh A có phần tử

nhỏ

Ta A có tính chất sau: dãy {am} chứa A amα thìαA Thật vậy, theo định nghĩa ta cóxmD chof(xm) =amα Vì dãy{xm}bị chặn (chứa D) nên có dãy con{xmk} hội tụ vềctrong

Rn Vì D đóng nên cDf(xm)α nên Vì f(xmk)→α Mặt khác, {xmk} → c f liên tục nên f(xmk) → f(c) Vì giới hạn

neân f(c) =α

Bây giờ, ta thấy A bị chặn (vì từ lập luận với α = −∞ ta

gặp mâu thuẫn) Do tồn tạiM =inf A Do định nghĩa inf tính chất

của A vừa trên, suy MA Vậy A có phần tử nhỏ M

(48)

Định lý mở rộng kết quen thuộc có SGK: "Cho [a, b] khoảng đóng R vàf : [a, b]→R liên tục, f có giá

trị nhỏ [a, b]" Do đó, mặt trực giác định lý rõ ràng

Tuy nhiên, bạn khó hình dung định lý liên quan đến vấn đề dồn biến? Hệ sau định lý "chìa khóa" cho định lý dồn biến Lưu ý tất kết mục không cần điều kiện f đối xứng

Định lý 2: Cho:

• D tập đóng, bị chận Rn, Λ tập đóng D

T :DD phép biến đổi

f :DR hàm số liên tục thỏa mãn f(x)> f(T(x)),xD

Thì ta có GTNN f đạt Λ, nghĩa là: f(x)>min

y∈Λ{f(y)},xD.

Chứng minh:

Do định lý 1, tồn x0D cho f(x0) ≤ f(x),xD Nếu x0

khơng thuộc Λ f(x0)> f(T(x0)), mâu thuẫn Vậy x0 ∈Λ ta có điều phải chứng minh

*Ghi chú: Ta thấy phép biến đổi T :DD thực định lý

trên địi hỏi tính chất T D\Λ Do với x ∈ Λ T(x) lấy giá trị tùy ý ta xem T(x) = x Quy ước

sử dụng phần lại, nghĩa T(x) =x,x ∈ Λ ta quan tâm giá trị T D

Đây hệ đơn giản phải không bạn, nhiên ý tưởng dồn biến lộ rõ Để minh họa, dẫn chứng minh cho BĐT Cauchy

Bài toán 1: (BĐT Cauchy) Chon số thực khơng âmx1, , xn Chứng minh

rằng:

x1+ +xnnn

x1 xn Chứng minh:

Bằng cách chuẩn hóa, ta giả sử x1 xn = chứng minh

x1+ +xnn Tất nhiên ta cần xét xin,i

Xét: D ={x= (x1, , xn)|xi ∈[0, n], x1 xn= 1}thì dễ thấy D đóng bị chặn Xét Λ ={x0 = (1,1,1, ,1)}

(49)

thì f(x) = x1 + +xn Xét T : D\Λ → D sau: Với x = (x1, , xn)D\Λ, tồn xi 6= xj ta đặt T(x) thu từ

x sau thay xi xj trung bình nhân chúng, dễ thấy

f(x)−f(T(x)) = (√xi

xj)2 >0

Vậy ta áp dụng định lí để suy raf(x)≥f(x0),xD,

dấu ” = ” xảy x=x0

Trong nhiều trường hợp, hàm f khơng đủ tốt ta

có có điều kiện f(x) ≥ f(T(x)) Tất nhiên ta khơng thể áp dụng

định lý Một địi hỏi hợp lý phép biến đổi T phải đủ tốt để bù lại (nhớ phép biến đổi T ta chọn) Điều đưa đến:

Định lý 3: Cho:

• D tập đóng, bị chận Rn, Λ tập đóng D

T : DD phép biến đổi cho tồn hàm số h liên tục DR thỏa mãn: h(T(x))< h(x),∀xD

f :DR hàm số liên tục thỏa mãn f(x)≥f(T(x))∀xD

Thì ta có GTNN f D GTNN f trênΛ, nghóa là: f(x)≥min

y∈Λ

{f(y)},xD.

Mặc dù trường hợp riêng định lý tổng quát cuối bài, tầm quan trọng định lý nên dẫn chứng minh cho

Chứng minh:

Lấy y0 ∈Λ cho f(y0) = y∈Λ

{f(y)} Giả sử phản chứng tồn zD cho f(z) < f(y0) Tất nhiên ta giả sử h(x) ≥ 0,xD

(nếu không việc thay h h0=hM, với M GTNN htrên D)

Chọn ε > đủ nhỏ ta có: f(z) +εh(z)< f(y0) Đặt g(x) := f(x) +εh(x), ∀xD Thì g : DR liên tục, g(x) > g(T(x))∀xDg(z) < f(y0)≤min

y∈Λ

{g(y)} Điều mâu thuẫn với định lý

Sau hệ ấn tượng định lý Hệ 1: (SMV-Strongly Mixing Variables) Cho:

DRn, D = {x = (x1, , xn)|x

iα,

P

xi = ns = const} vaø s0 := (s, s, , s)∈D

(50)

ai6=aj, thay ai, aj bới trung bình cộng chúng

f :DR hàm số liên tục thỏa mãn: f(a)≥f(T(a)),aD

Khi đó: f(a)≥f(s0),aD

Chứng minh: Với phép biến đổi T vậy, ta chọnh(x) =Pin=1x2i,x= (x1, , xn)D Áp dụng định lý (ở Λ ={s0})

*Nhận xét: Thông thường, áp dụng ta lấy ai, ajminmaxcủa

{a1, , an} Khi đó, chứng minh từ phần tử D, sau vơ hạn lần lặp T thu (s, s, , s), sử dụng tính liên tục củaf ta

thu kết luận

Riêng trường hợp (min max) khơng thiết thay ai, aj trung bình cộng mà tổng qt hơn:

Hệ 2: Cho:

DRn đóng bị chặn Gọi Λ tập hợp phần tử D có dạng

(s, s, , s), giả sửΛ khác rỗng

•Phép biến đổi T :DD sau: với phần tửa= (a1, , an)D\Λ,

ta chọn số i 6=j cho ai, aj minmax {a1, , an}, sau thay ai, aj α, β ∈(ai, aj).

f :DR hàm số liên tục thỏa mãn: f(a)≥f(T(a)), ∀aD

Khi đó:

f(x)≥min y∈Λ

{f(y)},xD.

Chứng minh: Một cách tự nhiên, ta hi vọng vào hàm

h(a) =max{a1, , an} −min{a1, , an},a= (a1, , an)D

Tuy nhiên, ta khơng có h(a) > h(T(a)), ∀aD\Λ Đó

n số a1, , an có nhiều số max hay min

{a1, , an} Nhưng ta việc thay T T∗ = Tn (Tk nghĩa lặp lại T với k lần) h(a)> h(T∗(a)),aD\Λ ta áp dụng định lý

Tuy nhiên, phép biến đổi T khơng đủ, ví dụ ta chưa biết xác dồn biến biên hay tâm Do đó, định lý mở rộng thành định lý dồn biến tổng quát sau

Định lý 4: (GMV − General Mixing Variables) Cho:

• D tập đóng, bị chận Rn, Λ tập đóng D

Tj : DD phép biến đổi cho tồn hàm số hj liên tục

(51)

f :DR liên tục thỏa mãn f(x)≥ j∈{1, ,k}

{f(Tj(x))},xD Thì f(x)≥min

y∈Λ

{f(y)},xD

Ta sử dụng lại chứng minh định lý với cải tiến nhỏ Chứng minh:

Laáy y0 ∈Λ cho f(y0) = y∈Λ

{f(y)} Giả sử phản chứng tồn zD cho f(z) < f(y0) Tất nhiên ta giả sử hj(x) ≥ 0,x

D,j = 1, , k Chọnε >0đủ nhỏ ta có: f(z)+εhj(z)< f(y0),j = 1, , k

Đặt gj(x) :=f(x) +εhj(x), ∀xD,j = 1, , k

Đặt g(x) = min{g1(x), , gk(x)},xD Thì g : DR liên tục, g(x) > g(T(x))∀xD\Λ vaø g(z) < f(y0) ≤

y∈Λ

{g(y)} Điều mâu

thuẫn với định lý

*Chi chú: Ta sử dụng kết gj hàm liên tục g =

min{g1, , gk} hàm liên tục Tất nhiên ta cần chứng minh với k = 2, trường hợp cần để ý min{g1, g2} =

1

2(g1+g2− |g1g2|) Cịn kiệng(x)> g(T(x)),xD\Λ rõ ràng,

uj > vj,j = 1, , k min{u1, , uk}> min{u1, , uk}

Các bạn thân mến, hình thức phát biểu ngắn gọn GMV có tầm ứng dụng rộng rãi Cứ (hay vài) phép biến đổi T thích hợp ta lại có định lý dồn biến Chúng kết thúc mục hệ GMV, mà xem mở rộng SMV Hệ Cũng xin lưu ý kết có tên SMV UMV tổng quát so với định lý tên mà dẫn ban đầu Hệ 3: (UMV− Undefined Mixing Variables) Cho:

D ⊂ {x= (x1, , xn)Rn|x

i ≥ 0,i= 1, , n}, D đóng bị chặn Gọi Λ tập hợp phần tử D có t thành phần nt thành

phần (t≥ 0)

•2 phép biến đổiT1, T2 :DD sau: với phần tửa= (a1, , an)D\Λ, chọn số i 6= j cho ai = min{at > 0, t = 1, , n}

aj = max{a1, , an}, sau thay ai, aj bới α, β ∈ (ai, aj) (ứng với T1)

α0 < a

i < aj < β0 (ứng vớiT2)

f :DR liên tục thỏa mãn: f(a)≥min{f(T1(a)), f(T1(a))}, ∀aD

Thì

f(x)≥min y∈Λ

(52)

Chứng minh:

Choïn h1(a) = max{a1, , an} −min{a1, , an} vaø h2(a) = −h1(a),

a = (a1, , an)D Tương tự hệ 2, ta thay T1 T1∗ = T

n

1 vaø T2∗ =T

n

2 để có: h1(a)> h1(T

1(a)), h2(a)> h2(T

2(a)), ∀a= (a1, , an)D

Áp dụng GMV ta có điều phải chứng minh

9 Nhìn lại.

Các bạn thân mến, có lẽ lúc tạm dừng để nhìn lại hành trình vừa qua Như chúng tơi nói trong$6, dồn biến biết đến từ sớm thông qua hàm lồi dẫn đến kết tuyệt đẹp BĐT Jensen xem tiêu chuẩn để dồn biến tâm cách tồn cục Về kết này, bạn tìm đọc cách đầy đủ "Bất đẳng thức" tiếng nhà toán học HardyP olyaLittewood

Trong trường hợp biến, có lẽ quen thuộc với bạn đọc BĐT lượng giác, chẳng hạn như:

sinA+sinB+sinC ≤ √

3

2 (1)

cosA+cosB+cosC

2 (2)

với A, B, C cạnh tam giác

BĐT (1) thu cách áp dụng BĐT Jensen cho hàm lồi BĐT (2) tinh tế hơn, hàm f(x) = −cosxf00(x) =cosx nên

lồi [0, π] Do ta khơng thể áp dụng BĐT Jensen cho biến

A,B,C Tuy nhiên, ta giả sử ABC A, B ∈ [0, π] nên ta dồn biến A, B Sau tốn cịn

biến trở nên đơn giản

(53)

Các bạn thân mến, dành mục để khảo sát vấn đề dồn biến cho BĐT biến để bạn nắm tư tưởng phương pháp, liệt kê tất kĩ thuật cần thiết Chẳng hạn dồn biến BĐT lượng giác với BĐT tuyệt đẹp Jackgarfulkel (xem phần tập) thú vị Tuy nhiên nghĩ trình bày tất nhàm chán vơ vị, nắm tư tưởng bạn áp dụng vơ vàn trường hợp khác

Đọc xong phần BĐT biến, có lẽ bạn đọc có cảm giác BĐT chuyển trường hợp biến biến đạt giá trị biên Phải nói điều cho hầu hết BĐT mà gặp Tuy nhiên, sau cung cấp cho bạn ví dụ nằm ngồi "thơng lệ" Trong ví dụ này, chí BĐT xét đa thức đối xứng biến Ví dụ lấy từ ý tưởng anh Bùi Việt Anh

Bài tốn 1. Cho a, b, c≥0 Khi BĐT:

(a3+b3+c3−6abc)2 + ((a+b+c)3−36abc)2 ≥0

chỉ xảy dấu "=" trường hợp (a, b, c) = (t,2t,3t), t≥0 (và hoán vị)

Bạn đọc tự kiểm tra điều

Như vậy, bạn yên tâm phương pháp dồn biến có ý nghĩa Với tốn biến thơng thường phải thực lần động tác dồn biến nên phức tạp Trong trường hợp n biến tổng qt việc dồn biến trở nên khó khăn Ngoài BĐT Jensen cho phép dồn lúc n biến (nhưng đáng tiếc, giải lượng nhỏ BĐT) gần ta khơng có cơng cụ khác Trong trường hợp này, thông thường quy nạp ý hay Chúng tơi dẫn ví dụ cho thấy tinh tế chứng minh quy nạp BĐT

Bài toán 2. (Phạm Kim Hùng) Cho n số thực dương a1, a2, , an có tích Chứng minh với k > thì:

1 (1 +a1)k +

1

(1 +a2)k + +

(1 +an)k ≥min{1,

n

2k}

(54)

làm việc với toán tổng quát hơn:

"Cho n số thực dươnga1, a2, , an có tích s≥1 Chứng minh với k > 0thì:

1 (1 +a1)k +

1

(1 +a2)k + +

(1 +an)k ≥min{1,

n

1 +√ns}.

Với toán tổng quát lại chứng minh quy nạp Thật vậy, xét toán với n số, ta giả sử an = min{a1, , an} Khi áp dụng giả thiết quy nạp cho (n−1) số a1, a2, , an−1 có tích ≥ 1, ta

đưa tốn biến Cơng việc lại khảo sát hàm biến

Một kĩ thuật khác để đưa BĐT n biến biến dồn biến

giá trị trung binh $7 Như ra, ý tưởng cách dồn dựa cách dồn biến giá trị trung bình cho hàm lồi Đây cách dồn biến tốt có tính hữu hạn Tuy nhiên, áp dụng cho cực trị đạt tâm

Bây ta phải đối mặt với khả cực trị đạt tâm biên Rõ ràng khả dồn biến khơng cao Do hi vọng vào điều tốt có cách dồn biến toàn cục, BĐT Jensen Với mục tiêu đó, định lý tuyệt đẹp phải kể đến định lý SMV (dồn biến mạnh) UMV (dồn biến khơng xác định) Hai định lý nói "anh em song sinh" SMV dùng để "chuyên trị" BĐT cực trị đạt tâm, cải tiến đáng kể khơng cần dồn biến mà cần dồn biến lớn biến nhỏ UMV địi hỏi giả thiết đặt lên biến bất kì, nhiên cho phép ta dung hịa trường hợp cực trị đạt tâm biên dạng tổng quát Để cho hình thức đơn giản, định lý xét cho hàm đối xứng

(55)

nhỏ nhất, dẫn đến cảm nhận rõ ràng n biến tiến giá trị trung bình, trường hợp bổ đề dãy số khơng cịn tác dụng Tuy nhiên, kết

Các bạn thân mến, bạn chúng tơi hành trình, mà chúng tơi chọn tốt khơng phải đầy đủ Có nhiều vấn đề chúng tơi khơng đưa ra, khơng trình bày kĩ, không coi trọng đầy đủ Cái mà coi trọng cố gắng để bạn thấy vấn đề cách nhanh chóng, rõ ràng hợp lý Hi vọng với tư tưởng mà khơi gợi bạn đủ cảm hứng khả để tiếp bước đường sáng tạo

Cuối cùng, muốn gửi lời cảm ơn đặc biệt tới anh Phan Thành Nam anh Phạm Kim Hùng, người có nhiều kết ý tưởng sử dụng Chúng xin chân thành cảm ơn tất tác giả tốn, nguồn trích dẫn, có thầy Phạm Văn Thuận −

người cung cấp cho chúng tơi tài liệu dồn biến có giá trị

10 Bài tập.

Sau số tập dành cho bạn đọc Hi vọng bạn tìm nhiều niềm vui thử sức với chúng (ghi chú: Θbài dễ, Φbài trung bình, Ξ khó, Ψ cực khó)

Θ Bài tập 1: (Asian Pacific Math.2004) Giả sử a, b, c số dương tùy ý

Chứng minh BĐT

(a2+ 2)(b2 + 2)(c2+ 2)≥9(ab+bc+ca)

Θ Bài tập 2: (MOSP 2001) Chứng minh a, b, c số dương có

tích ta có BĐT

(a+b)(b+c)(c+a)≥4(a+b+c−1)

Θ Bài tập 3: Choa, b, c không âm thỏa mãn a2+b2+c2 = Chứng minh

raèng

(56)

Θ Bài tập 4: (Huỳnh Tấn Châu) Cho x, y, zx+y+z = 1.Chứng

minh raèng:

x3+y3+z3+ 6xyz

4

Θ Bài tập 5: Chứng minh x, y, z số thực khơng âm thỏa

mãn điều kiện x2+y2+z2 = ta có BĐT: 7(xy+yz+zx)≤12 + 9xyz

Φ Bài tập 6: (Chọn đội tuyển Việt Nam 1996) Cho a, b, c số thực bất

kì, chứng minh rằng:

F(a, b, c) = (a+b)4+ (b+c)4+ (c+a)4−

7(a

4

+b4+c4)≥0

Φ Bài tập 7: (Phạm Văn Thuận−Zhao Bin) Giả sử x, y, z ba số thực

không âm có nhiều số Chứng minh

x3+y3 +

1

y3+z3 +

1

z3+x3 ≥

20 (a+b+c)3

Φ Bài tập 8 (Phạm Kim Hùng) Chứng minh với số thực a, b, c

không âm ta có BĐT:

4a2+bc+

1

4b2+ca+

1

4c2+ab

4

a+b+c

Φ Bài tập 9: (Murray Klamkin) Chứng minh với số thực không âm a, b, c có tổng 2,

(a2+ab+b2)(b2+bc+c2)(c2+ca+a2)≤3

Ξ Bài tập 10: (Tổng quát RMO2000) Cho a, b, c≥0và a+b+c= Tìm

hằng số k > 0nhỏ cho BĐT sau đúng:

ak+bk+ckab+bc+ca

(57)

Chứng minh rằng:

a2−bc+ 1 +

1

b2−ca+ 1 +

1

c2−ab+ 1 ≤3

Ξ Bài tập 12: (mathlinks) Cho a, b, c≥0 vàab+bc+ca= Chứng minh

raèng:

1 +a2b2

(a+b)2 +

1 +b2c2

(b+c)2 +

1 +c2a2

(c+a)2 ≥

5

Ξ Bài toán 13 Cho a, b, c∈[p, q] với 0< pq Tìm giá trị lớn của: a

b+c+ b c+a +

c a+b

Bài toán 14.(Jackgarfulkel) Cho tam giác nhọn ABC Chứng minh rằng: Φ a)

sinA

2 +sin

B

2 +sin

C

2 ≥

3(1 +sin

A 2sin B 2sin C 2) Φ b) cosA

2 +cos

B

2 +cos

C

2 ≥

3(1 +cos

A 2cos B 2cos C 2) Φ Bài toán 15.(Jackgarfulkel) Cho tam giác ABC Chứng minh rằng:

cos(AB

2 ) +cos(

BC

2 ) +cos(

CA

2 )≥

3(sinA+sinB +sinC) ΞBài toán 16 (Phan Thành Nam) Cho ba sỗ thựcx, y, zkhơng âm có tổng

bằng Chứng minh

p

x+y2+py+z2 +√z+x2≥ 2

Ξ Bài tập 17 (Vasile Cirtoaje) Xét ba số thực không âm a, b, c thỏa điều

kiện a2+b2+c2 = 1. Chứng minh rằng:

1 1−ab+

1 1−bc +

1 1−ca

(58)

ΞBài tập 18: (Phan Thành Nam) Choa, b, c≥0và thỏa mãna+b+c=

Chứng minh rằng: a) (VMEO1)

r

a+ (bc)

2

12 +

r

b+ (ca)

2

12 +

r

c+(ab)

2

12 ≤

3

b) p

a+k(bc)2+pa+k(bc)2+pa+k(bc)2 ≤√3

Trong k = 1−

3

Ξ Bài toán 19 (Phan Thành Việt) Cho tam giác ABC có độ dài cạnh

BC =a, CA=b, AB =c Gọi p nửa chu vi tam giác vàma, mb, mc độ dài ba đường trung tuyến tương ứng hạ từA, B, C xuống cạnh đối

diện Chứng minh rằng:

ma+mb+mc

r

3p2+

2[(ab)

2+ (bc)2+ (ca)2]

Bài tập 20: (Phan Thành Nam) Cho x, y, z ∈ [−1,1] vaø x+y+z =

Chứng minh

Ξ a) p

1 +x+y2+p1 +y+z2 + √

1 +z+x2 ≥3

Ψ b) r

1 +x+7 9y

2+

r

1 +y+ 9z

2+

r

1 +z+ 9x

2 ≥3

Φ Bài tập 21 (Phạm Kim Hùng) Cho x, y, z, t ≥ vaø x+y +z +t = 4.

Chứng minh rằng:

(1 + 3x)(1 + 3y)(1 + 3z)(1 + 3t)≤125 + 131xyzt

(59)

ΦBài tập 23(Phạm Văn Thuận−Nguyễn Anh Tuấn) Xét số thựca, b, c, d

thỏa mãn a2+b2+c2+d2 = 1.Chứng minh

1 1−ab+

1 1−bc +

1 1−cd +

1 1−da +

1 1−db +

1

1−ca ≤8

Ξ Bài tập 24 (Phạm Kim Hùng) Cho số thực không âm a, b, c, d, k

tổng 4. Chứng minh

(abc)k + (bcd)k + (cda)k+ (dab)kmax{4,(4 3)

3k

}

Ξ Bài tập 25 (Phan Thành Nam)

Cho số thực x, y, z, tthỏa: max{xy, yz, zt, tx} ≥1 Chứng minh

p

1−xy+y2+p1−yz+z2+√1−zt+t2+√1−tx+x2 ≥p16 + (xy+zt)2

Ψ Bài tập 26 (Phan Thành Nam) Cho số thực x, y, z, t ∈ [−1,1] thỏa mãn x+y+z+t= Chứng minh

p

1 +x+y2 +p1 +y+z2+ √

1 +z+t2 + √

1 +t+x2 ≥4

(*Ghi chú: Bài xuất phát từ trường hợp ba số 20a, dĩ nhiên

sẽ khó nhiều BĐT tương tự với số khơng cịn nữa)

Φ Bài tập 27 (Vasile Cirtoaje) Chứng minh a1, a2, , an khơng âm có tổng n

(n−1)(a21+a

2+ +a

n) +na1a2 ann2

Ξ Bài tập 28: (Phạm Kim Hùng) Giả sử a1, a2, , an số thực khơng âm có tổng n Tìm gtnn biểu thức

S=a21+a

2+ +a

n+a1a2 an(

a1 +

1

a2 + +

1

(60)

Ξ bài tập 29 Tìm số dương km tốt để BĐT sau với dãy số thực không âm x1, x2, , xn có tổng n

(1 +mx1)(1 +mx2) (1 +mxn)≤(m+ 1)n+km(x1x2 xn−1) m số dương

Bài tốn 30 (Phan Thành Việt) Cho a1, a2, , an, s, k số thực dương thỏa mãn: a1a2 an=snn−1 = (1+ns)k Xét BĐT:

1 (1 +a1)k +

1

(1 +a2)k + +

(1 +an)kn−1 Θ a) Chứng minh BĐT nói chung không

Φ b)(VMO 1999) Chứng minh BĐT trường hợp k =

Ngày đăng: 06/02/2021, 20:15

Từ khóa liên quan

Tài liệu cùng người dùng

  • Đang cập nhật ...

Tài liệu liên quan